You are on page 1of 96

OPEN UNIVERSITY

of MAURITIUS

Financial
Theory & Practice
OUBS002124

OPEN UNIVERSITY
of MAURITIUS

Financial
Theory & Practice
OUBS002124

Open University of Mauritius - Financial Theory & Practice

PROJECT COORDINATION
Open University of Mauritiius

Open University of Muritius, May 2013


First published 2013

All rights reserved. No part of this work may be reproduced in any form or by any means, without
prior written permission from the Open University of Mauritius. Commercial use and distribution of
this material is strictly prohibited.

ii

Open University of Mauritius - Financial Theory & Practice

OPEN UNIVERSITY
of MAURITIUS

Financial
Theory& Practice
OUBS002124

Table of Contents
UNIT 1 - THE FINANCIAL SYSTEM

UNIT 2 - CAPITAL MARKETS

11

UNIT 3 - THE TIME VALUE OF MONEY

21

UNIT 4 - CAPITAL INVESTMENT APPRAISAL

33

UNIT 5 - SHARE VALUATION, RISK AND RETURN

45

UNIT 6 - FIXED INCOME SECURITIES

55

UNIT 7 - RISK, RETURN AND DIVERSIFICATION

69

Assignment 83
SOLUTIONS

Open University of Mauritius - Financial Theory & Practice

85

Open University of Mauritius - Financial Theory & Practice

UNIT

THE FINANCIAL SYSTEM

Unit Structure
1.0 Overview
1.1 Learning Objectives
1.2 Introduction
1.3 Functions of the Financial System
1.4 Financial Intermediaries
1.5 Primary and secondary markets
1.6 The Relationship between Banks and Companies
1.7 Summary
1.8 Tutorial
1.9 Suggested Reading

Open University of Mauritius - Financial Theory & Practice

1.0 OVERVIEW
This unit lays out the foundation to the understanding of the importance of financial
markets to the economy. The chapter will enable students to appreciate the various
aspects and concepts of financial markets.

1.1 LEARNING OBJECTIVES


By the end of this Unit, you should be able to:
1. Define a financial market and explain its importance to the economy.
2. Explain the role of financial intermediaries.
3. Discuss the differences between the primary and secondary markets.
4. Describe the importance of the banking system to companies

1.2 INTRODUCTION
A financial market helps transfer financial assets, real assets and financial risks in
various forms from one entity to another, from one location to another and across time.
The central figure to the whole world of finance is the management of cash as depicted
in Figure 1.
Shareholders

Equity Capital

Employees

Loan
Capital

Lenders

Cash

Government

Government

Customers

Suppliers of Raw
Materials
Figure 1: Uses of Cash

The two main sources of cash are from equity capital and loan capital.

(a) Equity Capital

Companies raise money by issuing equity (shares) that is turn bought by individual
and institutional investors. This capital is then invested in various projects that
will bring added profitability to the company. When investors buy a share in a
particular company, they become entitled to future dividends, paid out of profits
by the firm. These investors are also entitled to participate in the management of
the company through their votes in Annual General Meetings (AGMs).

(b) Loan Capital

The public, companies and government very often need to spend money now, but do
not have sufficient funds. This loan capital can be obtained by contract a loan with
their banks or other financial institution. The loan is then serviced by meeting interest
payment and capital repayment within the time frame as agreed by both parties.
The Government can also borrow money from investors for various reasons. An
example of the debt contracted by the Government of Mauritius, through the Bank
of Mauritius, is depicted in Illustration 2. Such debts are referred to as Government
Treasury Bills since they have a maturity of less than a year (182 days). Treasury
Notes have a maturity that ranges from 1 year to 10 years. Treasury bonds have
a maturity exceeding 10 years. The Government repays those debts through the
taxes that they collect and from the projects funded by these debts. The following
is an example of a treasury bill issued by Mauritius.

Open University of Mauritius - Financial Theory & Practice

Box 1: Notice of Tender for Government of Mauritius Treasury Bills


Government of Mauritius Treasury Bills (GMTB) with maturity of 182 days for a
nominal amount of Rs800million will be sold through auction on Friday 08 March
2013, for same day settlement to:
(i)
Primary Dealers, and
(ii) Exceptionally and until further notice, to Non-Primary Dealer banks and
other eligible Financial Institutions, which have participated in the primary
auction held by the Bank at least twice fortnightly.
The Bank will receive bids for this auction on Friday 08 March 2013 on a yield
basis quoted to two decimal places, in multiples of Rs100,000, on tender forms
which are obtainable at its seat or on its website.. Applications received after the
prescribed time and date will not be considered.
Banks may submit bids through the Reuters Dealing System.
In the event of oversubscription of the GMTB, the Bank may issue Bank of
Mauritius Bills of the same tenor to bidders who are willing to so accept at the
weighted accepted yield for above maturity of GMTB.
Results of the auctions will be announced on the same day and successful bidders
will be required to effect payment of the cost price of the GMTB/Bank of Mauritius
Bills through the Mauritius Automated Clearing and Settlement System.
Adapted from the Bank of Mauritius
Source: Bank of Mauritius
Normally, a company in need of capital will use the services of a financial intermediary
i.e. a bank to get access to the required funds. Alternatively, companies listed an
exchange may opt to float their debt directly on the market. Through such an undertaking,
companies lower their cost of debt by paying a lower rate of interest since they have
omitted the use of the bank.
Consider the example of Omnicane in Box 2 where the company wished to contract debt
directly from potential investors on the Stock Exchange of Mauritius. Box 2 showcases
the Note issued by Omnicane. The company will pay interest semi-annually at the rate
of 5.70% and the debt will be repaid on the 17th January 2016.

Open University of Mauritius - Financial Theory & Practice

Box 2: Omnicane Limited


Multicurrency Medium Term Note Programme
(Aggregate Maximum amount: Up to MUR 3 Million)
Second Tranche of up to MUR 920Millions
Further to the communiqu dated 28 November 2012, Omnicane Limited hereby
announces the results of the auction for the second issue of notes in respect of the
above and held on 05 December 2012 as follows:
Issuer

Omnicane Limited

Aggregate Nominal Amount Required MUR 920,000,000


Nominal Amount per Note

MUR 100,000

Bids Received

MUR 1,000,000,000

Bids Accepted

MUR 920,000,000

Clearing Rate

5.70%

Fixed Interest Rate payable

5.70% p.a. payable semi-annually in


arrears on 18 July and on 18 January
of each year until maturity

Settlement Date

18 January 2013

Issue Date

18 January 2013

Commencement of Trading

21 January 2013

Maturity Date and Redemption Date 17 January 2016


On the first day of trading of the second tranche of Notes issues, scheduled for the
31 Jnuary 2013, a minimum of 10 Notes will be made available for trading.
Source: Omnicane Limited

1.3 FUNCTIONS OF THE FINANCIAL SYSTEM


The two main uses of the financial system are raising equity from stock markets and
borrowing from the financial markets. The other uses of the financial system include:

(a) Saving money for future consumption

People always save a portion of their salaries for consumption in future periods.
E.g. Workers who save for their retirement needs move some of their actual
earnings into the future. At retirement, they then use these savings to cater for
their needs. To move money across time, savers have a host of products in which
to invest. Investors need a fair rate of return to compensate them for the use of
their money and in the event that they lose the money. Therefore, a conservative
investor will prefer to invest in Government Treasury Bonds as these as being risk
free. A risk seeking investor will invest in other financial products such as mutual
funds, stocks or even banks. Given the riskiness of these financial instruments, the
return on each of them will differ, with the riskier asset offering more return.

Open University of Mauritius - Financial Theory & Practice

Activity 1
Suppose you are a financial analyst working for a well-known bank. Currently,
the market is offering the following rates:
Instrument

Return

Banks savings rate

5%

Government Treasury Bond

4%

Mutual Fund (Expected)

7%

Stock A (Expected)

10%

Stock B (Expected)

12%

During the day, two potential investors take an appointment with you to discuss
their financial situation and to identify suitable investment opportunities.
Both investors have a capital of Rs 1million to invest.
The first investor is aged 30 and from his personality and subsequent
discussion with him, you find that he is a risk seeking individual. The second
investor has just retired from service and is looking for an investment that
will allow him to have access to the funds at any point in time. Return is not
of a major concern to him and you determine that he is a risk-averse investor.
What are the investments that you will recommend to each of the investors?

(b) To exchange assets for immediate and future deliveries

Individuals and companies frequently trade one asset for another based on their
usefulness to them at a particular point in time. Investors may trade one currency for
another one or use money to buy a commodity, etc. Consider the following example:

- A Mauritian hotel company e.g. LUX* Resorts pays its Mauritian workers in
Rupees but invoices its services to tourists in Euros. Therefore, it will need to
trade in the foreign exchange markets to exchange Euros for Rupees.

(c) To manage risks

Investors face financial risks which include default risk, a change in foreign
exchange rates, a hike in raw materials, amongst others. These risks can be hedged
by trading different contracts such as forward contracts, futures contracts, swaps,
option contracts, etc. Consider the following:

- Arabica and a coffee manufacturer face different risks related to the price of
coffee. Arabica fears that the price of coffee will increase while the coffee
manufacturer believes that the price of coffee will fall. They can both eliminate
their exposures by entering into a binding agreement referred to as a forward
contract. In such a contract, the coffee manufacturer agrees to sell a fixed quantity
of coffee to Arabica for a specified price and under the contract, Arabica is
obligated to buy the coffee at that specified price. Such an undertaking therefore
eliminates both parties exposure to changing coffee prices.

(c) P
 rice discovery of assets at very low costs, thereby aiding liquidity of
financial products trading in different markets

Stock prices and interest rate are valuable information used by different investors
as to their pattern of consuming, saving or dividing their capital among different
asset classes. However, research of these information is time-consuming and
a costly one for an individual investor. However, when a financial institution
researches the market price/rates, the cost is greatly reduced. The fact that such
information is available at very low cost makes trading easier and hence increases
the liquidity of different assets across different markets.
Open University of Mauritius - Financial Theory & Practice

1.4 FINANCIAL INTERMEDIARIES


Financial intermediaries have developed to bring lenders and borrowers together
and help these entities to achieve their financial goals. These financial intermediaries
include banks, credit unions, brokers, exchanges, dealers, mutual funds, hedge funds,
insurance companies and various other finance corporations. Financial intermediaries
are fundamental to the smooth running of the financial system.
Brokers are responsible for executing orders on behalf of their clients. Their mission is
to identify traders who are willing to take the opposite side of their clients orders.
Investment banks provide advice to corporate clients and their expertise lies in
structured finance, mergers and acquisitions, initial public offerings.
Exchanges are platforms where traders meet to arrange and execute their trades.
Historically, exchanges used to be physical locations where buyers and sellers would
meet. Over time, it has evolved to electronic exchanges whereby all transactions are
executed over computer terminals. Examples of exchanges are NYSE-Euronext,
Tokyo Stock Exchange, NASDAQ, Chicago Mercantile Exchange and London Stock
Exchange.
Dealers, unlike brokers, fill their clients order by trading with them. After executing
this initial transaction, dealers will then try to reverse such transactions by entering into
an opposite trade with another client and making a profit. Such transaction enhance the
liquidity of a particular market and hence, of financial products.
Mutual Funds or investment companies provide the added advantage of pooling and
managing the money of various investors. Individual investors portfolios are not large
enough to be able to buy the universe of securities compared to a mutual fund as this
would tantamount to holding a number of individual stocks. On the other hand, the
mutual fund offers the benefit of economies of scale since the transaction costs incurred
whilst buying the stocks is dispersed over a much higher portfolio size.
Insurance companies help investors to minimize risks by creating insurance contracts
(policies) against a payment in the even that some loss occurs. These contracts thus
provide a hedge against potential losses. Examples of insurance contracts include,
automobile, fire, life, medical, theft, etc.

1.5 PRIMARY AND SECONDARY MARKETS


Once a security has been issued, it lives a life of its own, in that it is sold from one
investor to the other. The life of the security is connected to the fact that it can be bought
or sold at any moment. For example, shares that were created when a company was
incorporated can later be floated on an exchange. New issues of a security are floated on
a primary market. Subsequent transactions, e.g. shares of the company being bought
or sold, occur on a secondary market.
The main difference between the primary and secondary market is that the primary
market is used for the issue of new financial products, from equity issues to bond
issues. On the other hand, the secondary market is the market for used financial
products. Instruments bought and sold on this market must have already been created
and are simply exchanging hands, without any new security being issued.
In essence, the task of the secondary market is to ensure that the financial products are
correctly priced and traded, thus increasing the liquidity of the product. The difference
between the primary and secondary market is conceptual; they are not physically
separated from each other.
8

Open University of Mauritius - Financial Theory & Practice

Secondary markets play a vital role in valuing securities. Any investor buying a certain
financial instrument does not wish to remain invested in that particular asset indefinitely.
Consider the case of a share and a bond. An investor, who has bought a share at a price
of Rs100, will want to realize a profit when the shares price increases to Rs150. Hence,
the importance of a secondary market to provide liquidity (ability to buy or sell a product
in a relatively short period of time and without incurring any loss of value) to sell the
share at Rs150. Comparatively, a bond has a maturity date and a certain redemption
value (price that will be repaid at the end of the maturity period). If an investor holding
a bond wished to hold onto the bond until maturity, he may do so. In other cases, should
he wish to sell the bond before the maturity date, he may do so if a liquid market for
the product exists such that the market will value the security at a certain price at that
particular date.

1.6 THE RELATIONSHIP BETWEEN


BANKS AND COMPANIES
Bank intermediation is provided mostly by commercial banks. The latter serve as
intermediaries between those having a surplus of funds and those that in need of capital.
The business of banking is such that these institutions collect capital from the former
and lend to a wide range of borrowers that include individuals, companies and even the
Government.
The risk of lending is borne by these banks and their balance sheet should be adequately
funded and the portfolio of clients diversified enough to minimize the credit risk and
hence, systemic risk on other banks and the economy should massive defaults arise in
the economy.
Over the years, commercial banks have provided a wide range of value added services to
their customers so as to provide them with funds in a more efficient and timely manner.
Banks have further devolved and thus helped their corporate clients gain direct access
to capital markets, leading to the rise of investment banking. The latter includes, but is
not limited to, the following services:
Access to Equity Markets: Investment bankers help companies to list their shares
on the stock market. They are responsible for preparing the initial public offering
process such as supporting their client to prepare a prospectus. Later on, they
advise the company on additional issues that the firm may require and on the type
of instrument that is best suited for the issue.
Access to Bond Markets: Just like shares, investment banks can also help firms to
list their debt on a bond market. Refer to Box 2 The case of Omnicane Limited.
Mergers and Acquisitions: Investment bankers are in contact with leading directors
and are always on the lookout for potential mergers and acquisitions so as to help
these firms to grow even further and accentuate their dominance on the market.
Asset Management: Banks use their deep knowledge of financial markets to
provide this additional service to individuals, companies and other institutions.
The products on offer comprise portfolios of listed and unlisted securities, bonds,
commodities, real-estate. These portfolios are referred to as mutual funds.

Open University of Mauritius - Financial Theory & Practice

1.7 SUMMARY
The importance of the financial system to the economy has thoroughly been discussed
in this unit. The key players in the form of financial intermediaries play an important
role and are key to the effective transmission of services and capital. Lastly, we oversaw
the differences between the primary and secondary markets and shedding light on the
relationship between firms and the banking system.
The next unit shall be geared towards capital markets (Stock markets, bond markets,
money markets, derivatives markets, futures markets, forward market, commodities
market, real estate market) and the different financial products traded on those markets.
Close attention shall be paid to the market existing on the Mauritian landscape.

1.8 TUTORIALS
Question 1

How can households, who need to invest their excess funds, reduce their risk?

Question 2

Financial intermediaries can basically be split in two: brokers and market makers.
(a) Discuss the differences between brokers and market makers
(b) In your opinion, should the price earned by market makers be at a premium to the
commission earned by brokers?

Question 3

The evolution of the financial system has contributed massively to the growth of various
economies around the world. Discuss the ways in which the financial system has aided
in this growth with the use of suitable examples.

Question 4

With the use of relevant examples (i.e. recently issued stocks and already trading stocks),
explain the differences existing between the primary and secondary markets?

Question 5

Central Banks across the world have inferred that companies are increasingly dependent
on banks to fund future growth and this relationship has since been under close
supervision due its repercussion on the economy during times of crisis. Discuss the
intricacies of the working relationship between firms and the banking system.

1.9 SUGGESTED READINGs


Vernimmen, Pierre; Quiry, Pascal; Dallocchio, Maurizio; Le Fur, Yann; Salvi, Antonio,
Corporate Finance, John Wiley & Sons, Ltd, Second Edition (Chapter 1)
Brealey, Richard A.; Myers, Stewart C., Principles of Corporate Finance, Latest Edition
(Chapter 1)
Pike, Richard; Neale, Bill, Corporate and Finance Investment Decisions and Strategies,
Second Edition (Chapters 1 and 2)
Bodie, Zvi; Kane, Alex; Marcus, Alan J., Investments, Eight Edition (Chapter 1)

10

Open University of Mauritius - Financial Theory & Practice

UNIT

CAPITAL MARKETS

Unit Structure
2.0 Overview
2.1 Learning Objectives
2.2 Stock Markets
2.2.1 Initial Public Offering
2.2.2 International stock market indexes
2.2.3 The Stock Exchange of Mauritius and the Development Enterprise Market
2.2.4 Measuring the return of a stock
2.3 Bond Markets
2.4 Money Markets
2.5 Derivatives Markets
2.5.1 Futures Market
2.5.2 Forward Market
2.5.3 Swap Market
2.5.4 Options Market
2.6 Commodities Markets
2.7 Real Estate Market
2.8 Summary
2.9 Tutorial

Open University of Mauritius - Financial Theory & Practice

11

2.0 OVERVIEW
After the brief introduction from the previous chapter, Unit 2 explores the world of equities
in detail, spanning from the Mauritian Stock Exchanges to worldwide exchanges. Our
attention to then shift to fixed income markets (bond and money markets). Derivatives
markets (futures, forwards and swaps). An emerging investment trend in the form of
commodities and real-estate is also introduced here.

2.1 LEARNING OBJECTIVES


By the end of Unit 2, you should be able to:
1. Appreciate how stocks are issued on the stock market and able to discuss about
major stock exchanges
2. Differentiate between bond markets and money markets
3. 
Assess the basic characteristics of futures contracts, forward contracts, swap
agreements and options markets.
4. Discuss alternative investment forms commodities and real estate markets

2.2 STOCK MARKETS


A stock exchange has two principal economic functions. These are to enable companies
to raise new capital (via the primary market) and to facilitate the trading of existing
shares (via the secondary market) through the negotiation of a price at which title
to ownership of a company is transferred between investors. Table 1 presents some
stock markets.
Country

Exchange

Mauritius

Stock Exchange of Mauritius

United Kingdom

FTSE

France

CAC 40

Germany

Frankfurt Stock Exchange

U.S.

New York Stock Exchange

India

Bombay Stock Exchange

China

Shanghai Stock Exchange

South Africa

Johannesburg Stock Exchange


Table 1: Various Stock Exchanges around the World

Box 1: What is a Share?


There exist basically two types of shares: Common shares and Preference shares.
Common Shares represent an ownership or stake in a company and are the leading
type of equity security. On purchase of a common share in a company, the shareholder
is entitled to participating in the governance of the firm through voting rights at
Annual General Meetings (AGMs). These shareholders are also entitled to a claim on
the companys assets in the case of liquidation. Companies may pay out all of their
profits in the form of dividends to shareholders, but they are obligated to so.
On the other hand, holders of preference shares do not participate in the operating
performance of the company nor do they have any voting rights. However, preference
shares rank above common shares with respect to the payment of dividends and the
distribution of the firms net assets in the event of liquidation. Dividends paid on
preference shares are inherently fixed, similar to interest payments.
12

Open University of Mauritius - Financial Theory & Practice

2.2.1 INITIAL PUBLIC OFFERING

There are two types of primary market issues of common stock.


a) Initial public offerings or IPOs are stocks issued by a formerly privately owned
company that is going public, that is, selling stock to the public for the first time.
b) Seasoned equity offerings are offered by companies that already have floated
equity. For examples, a sale by IBM of new shares would constitute a seasoned new
issue. (Bodie, Kane, & Marcus, 2009)
Investment bankers normally manage the issue of new securities to the investing
community. Once the Financial and Services Commission has given its approval, the
investment bankers can distribute a prospectus and organize road shows to market the
stock and the company. These road shows are important in that it helps to generate
interest about the company and provide financial and non-financial information about
the firm to investors. The latter can also formulate the price at which they would like to
purchase the stock of the company. These indications of interest are referred to as book
and the process of polling potential investors is referred to as bookbuilding.
Box 2: Facebook IPO Facts, Fiction and Flops
Facebook Inc.s fiasco is still the talk of Wall Street. The newly public shares are
losing an average of about $1 per trading day since their offering. If that lasts, the
social-networking company would be worth nothing before the end of June, and
Chief Executive Mark Zuckerbergs trips to McDonalds will seem less chic and
more necessary.
Maybe the biggest miss is this: Facebook was a big bomb.
That might be true when measured against all the hype Facebook got leading up to
its first day of trading. When it comes to the U.S. market for initial public offerings,
though, Facebook was just a high-profile flop in a series of lesser-known flops.
Overlooked is that Facebook decided to make its entrance into an U.S. IPO market in
the dumps and last year wasnt a blockbuster for U.S. IPOs. Even though IPOs worldwide are rising by 43% on the first day of trading, first-day gains arent as common in
the U.S. Of the 53 U.S. deals priced in the last three months, the average price increase
was 14% on the first day of trading. But through May 25, the average increase over the
offering price was 8% for IPOs this year.
Like Facebook, many initial public offerings were overvalued by traditional measures.
They simply have to come down to earth. Many of the companies that have seen sharp
drops from their offering prices were trading at multiples that overestimated future
growth: price-to-earnings ratios of 20, 30 or more.
Facebook trades at 60 times trailing 12-month earnings, 49 times prospective 12-month
earnings, 43 times cash flow and 12 times book value. A report by Thomson Reuterss
concluded that on a fundamental basis it is worth only about $10.22 a share.
So was Facebooks IPO a flop? Yes. But it was only a flop in a time when most
offerings didnt bother to risk the embarrassment.
It is one thing to disappoint compared to other IPO newborns and quite another to
compare to the stillborn public corporation.
By another measure, Facebook was far from a failure. The investors who cashed out
at the top are hardly complaining. They maximized their take through the efforts of
Morgan Stanley and the underwriters who marketed Facebook.
In the end, they paid for an IPO to make them rich. It was up to the rest of us to buy it.
Source: Wall Street Journal (31st May 2012)

Open University of Mauritius - Financial Theory & Practice

13

2.2.2 International Stock Market Indexes

For one particular country, there may exist various exchanges. Any exchange may
possess one index or many indexes. A stock market index is a technique of measuring the
value of a stock market or only a section of the stock market. An index is an important
tool for investors and finance professionals alike in that it describes the evolution of the
market over a certain time period.
Many stock exchanges exist in the U.S. for e.g. NYSE, NASDAQ, S&P500, etc.
Different indices proposes various ways of following these markets e.g. the Wilshire
5000 index computes the market value of all NYSE and American Stock Exchange
including actively traded NASDAQ stocks. On the other hand, the Dow Jones or DJIA
includes 30 blue-chip companies. This index has been calculated since 1896.

2.2.3 Stock Exchange in Mauritius

The stock exchange of Mauritius operates two markets: the Official Market and the
Development and Enterprise Market. There are 42 companies listed on the SEM and
47 companies listed on the DEM. For a company to be able to list on the SEM or the
DEM, it needs to satisfy the exchanges listing requirements. The Official Market of the
SEM has three major indexes: SEMDEX, SEM-7 and SEMTRI. On the other hand, the
indexes of the DEM include the DEMEX and the DEMTRI.

Activity 1

Using the official website of the Stock Exchange of Mauritius, find the following
information:
(a) Under which groupings has companies listed on the SEM and the DEM been
categorized?
http://www.stockexchangeofmauritius.com/officialmarket-listedcompanies
(b) W
 hat are the listing requirements for a company wishing to list on the SEM
and the DEM?
http://www.stockexchangeofmauritius.com/listingrules
http://www.stockexchangeofmauritius.com/dem-listingrules
(c) W
 hat are the differences between the SEMDEX, SEM-7 and the SEMTRI?
http://www.stockexchangeofmauritius.com/officialmarketindices/index/
semdex/weekly#
(d) W
 hat are the difference between the DEMEX and the DEMTRI?
http://www.stockexchangeofmauritius.com/officialmarketindices/index/
semdex/weekly#
Note:
The official website of the stock exchange of Mauritius is:
http://www.stockexchangeofmauritius.com/

2.2.4 Measuring Return

A stock market index is used to reflect changes in the average value of a list of
companies which are traded on a particular market. For example, in France the CAC 40
provides such a means. The same goes for the Dow Jones in the U.S. and the SEMDEX
in Mauritius.
The market capitalization is used to measure the size of a company or the size of the
stock market or the size of part of the stock market.
Market Capitalisation of Company A

= (Number of shares of Company A) X (Price of a share of Company at time t)

14

Open University of Mauritius - Financial Theory & Practice

Consider the following example:


An analyst gathers the following information for a market-capitalisation-weighted
index comprised of securities A, B and C. The aim of this exercise is to find the total
return of the index.
Beginning of
Period Price/
MUR

End of Period
Price/MUR

Dividends Per
Share/ MUR

Shares
Outstanding

2,500

2,700

100

5,000

3,500

2,500

150

7,500

1,500

1,600

100

10,000

Security

The Beginning of Period Price column indicates the price of a share at the beginning
of the period under investigation.
The End of Period Price column indicates the price of a share at the end of the period
under investigation.
The Dividends Per Share column indicates the amount of dividends paid to the holder
of one share.
The Shares Outstanding column shows the number of shares held by the company.
Return of Stock A over the period considered

= (Price at end Price at beginning / Price at beginning)

= (2700-2500 / 2500) * 100%

= 8%
Total Return of Stock A over the period considered
= Price at end + Dividends Paid Price at Beginning
Price at Beginning

= (2700 + 100 2500) / 2500

= 12%

Carrying out the same calculation for stock B and C will yield:
Stocks

Simple Return

Total Return

-28.6%

-24.3%

6.7%

13.3%

Total Market Capitalisation of Stock A



= No. of Shares of A X Price of Share of A

= 5,000 X 2,700

= MUR 13,500,000
Total Market Capitalisation of Stock B

= No. of Shares of B X Price of Share of B

= 7,500 X 2,500

= MUR 18,750,000

Open University of Mauritius - Financial Theory & Practice

15

Total Market Capitalisation of Stock C



= No. of Shares of C X Price of Share of C

= 10,000 X 1,600

= MUR 16,000,000
Total Market Capitalisation of the Index

= Total Market Capitalisation of Stock A + Total Market Capitalisation of Stock B

+ Total Market Capitalisation of Stock C

= 13,500,000 + 18,750,000 + 16,000,000

= MUR 48,250,000
Weight of a stock in the index

= Market Capitalisation of the stock / Market Capitalisation of Index
Market
Capitalisation of
the stock / MUR

Total Market
Capitalisation of
Index/ MUR

Weight / %

13,500,000

48,250,000

27.98

18,750,000

48,250,000

38.86

16,000,000

48,250,000

33.16

Weight

Return of Index = Weight of a particular Stock X Total Return of the Stock


Weight / %

Return of the
Stock / %

Weighted Return

27.98

12%

3.4%

38.86

-24.3%

-9.4%

33.16

13.3%

4.4%

Return of Index

100%

-1.7%

Hence, the return of the index is -1.7%

2.3 Bond Markets


The bond market comprises longer term debt instruments other than those that trade on
the money market. This market includes Treasury notes and bonds, corporate bonds,
municipal bonds, amongst others. Most of these instruments promise either a fixed stream
of income or a stream of income that is determined according to a specific formula. It is
common practice to refer to these instruments as debt instruments or bonds.
Treasury notes and bonds are issued by the Government and are virtually considered
as being risk free i.e. there is no risk that the Government will default on repayment.
Treasury notes have a maturity of up to 10 years while Treasury bonds have a maturity
greater than 10 years. These debt instruments pay annual or semi-annual coupons over
the life of the bond, with final repayment at maturity. (Refer to Chapter 1 Box 1)
Corporate bonds are issued when firms borrow money directly from the public. These
bonds are listed on an exchange and are typically similar to Treasury notes and bonds in
that they pay annual or semi-annual coupons over the life of the bond and return the face
value to the holder of the bond at maturity. (Refer to Chapter 1 Box 2)
Municipal bonds are issued by state and local governments. They are similar to treasury
securities exempt that their interest income is exempt from tax. The interest income
is also exempt from state and local taxation in the issuing state. Capital gains tax is
however payable when the bonds mature or are sold prior to their maturity dates.
16

Open University of Mauritius - Financial Theory & Practice

Convertible Bonds give the bondholder a guaranteed coupon as well as the option
to exchange the bond into a predetermined number of common stocks of the issuing
company. While this feature allows investors to take advantage of favourable movements
in the price of the issuing companys common stocks, it also enables issuers to offer a
lower coupon rate and thereby reducing their cost of financing. However, in the event of
a default by the issuing company, both the bond and the conversion option may become
worthless. Furthermore, the company can also force conversion by calling the bond.
Eurobonds are international bonds which can be issued by both corporations and
governments with the main aim of tapping secured and long-term financing across a
geographical diverse investment clientele. It is generally underwritten by an international
syndicate and sold simultaneously in many countries different from the country of the
currency in which the issue is denominated. Thus, a dollar-denominated Eurobond
would be sold outside the United States only. As an example, a Greek corporation
issuing dollar-denominated bonds through a consortium of Japanese, Greek and UK
investment banks.

2.4 Money Markets


The money market is a sub-sector of the fixed income market. It consists of very
short term debt securities that usually are highly liquid. These securities trade in large
denominations and are out of reach of individual investors. Examples of instruments
traded on money markets include Treasury Bills, Certificates of Deposits, Commercial
Paper, amongst others.
Treasury Bills are most liquid and marketable of all money market instruments.
Investors buy bills at a discount from the stated maturity value. At the bills maturity,
the holder receives from the government a payment equal to the face value of the bill.
The difference constitutes the investors gain. T-Bills are issued with initial maturities
of 28, 91, 182 days. Individuals and institutional investors can purchase T-Bills directly
at auction or on the secondary market from a government securities dealer.
A certificate of deposit or CD is a time deposit with the bank. Time deposits may not be
withdrawn on demand. The bank pays interest and principal to the depositor only at the
end of the fixed of the CD.
Commercial papers are short-term unsecured debt notes that are issued by large and
well-known companies. Commercial paper maturities range up to 270 days and are
considered to be fairly safe assets since a firms condition presumably can be monitored
and predicted over a term as short as 1 month.

2.5 Derivative Markets


One of the most significant developments in financial markets in recent years has
been the growth of futures, options and related derivatives markets. These instruments
provide payoffs that depend on the values of other assets such as commodity prices,
bond and stock prices or market index values.

2.5.1 Futures Markets

A Futures contract calls for delivery of an asset at a specified delivery or maturity date
for an agreed-upon price, called the futures price, to be paid at contract maturity. The
long position is held by the trader who commits to purchasing the asset on the delivery
date. The trader who takes the short position commits to delivering the asset at contract
maturity. The long position, which commits to purchasing, gains if the asset value
increases while the short position, which commits to selling, loses.

Open University of Mauritius - Financial Theory & Practice

17

Unlike a forward contract, however, a futures contract is not a private and customized
transaction but rather a public transaction that takes place on an organised exchange. In
addition, a futures contract is standardised the exchange, rather than the parties, sets
the terms and conditions, with the exception of price. Also, parties to futures contracts
are guaranteed against credit losses resulting from the counterpartys inability to pay. A
clearing house ensures that credit risk is eliminated.

2.5.2 Forward Markets

A forward contract is an agreement between two parties in which one party, the buyer
agrees to buy from the other party, the seller, an underlying asset or other derivative, at
a future at a price established at the start of the contract. The global market for forward
contracts is part of a vast network of financial institutions that make markets in these
instruments. Transactions in forward contracts typically are conducted over the phone.

2.5.3 Swap Markets

Although swaps were the last of the main types of derivatives to be invented, they are
clearly not the least important. In fact, judging by the size of the swap market, they are
probably the most important. The Bank of International Settlements had estimated the
notional principal of the global over-the-counter derivatives market as of 30th June 2001
at $100 trillion. Of that amount, interest rate and currency swaps accounted for about $61
trillion. Swaps are widely used by corporations, financial institutions and Governments.
Swaps are multi-period extensions of forwards contracts. For example, rather than
agreeing to exchange British Pounds for U.S. dollars at an agreed-upon forward price
at one single date, a foreign exchange swap would call for an exchange of currencies
on several future dates. Similarly, interest rate swaps calls for the exchange of a series
of cash flows proportional to a given interest rate corresponding series of cash flows
proportional to a floating interest rate.

2.5.4 Option Markets

A call option gives its holder the right to purchase an asset for a specified price, called the
exercise price or strike price, on or before a specified expiration date. For example, a December
call option on a MCB share with an exercise price of MUR 35 entitles its owner to purchase
the MCB stock for a price of MUR 35 at any time up to and including the expiration date in
December. The holder of the call need not exercise the option; it will be profitable to exercise
only if the market value of the asset that may be purchased exceeds the exercise price. On the
other hand, a put option is the right to sell an asset at some exercise price. Calls increase in
value while puts decrease in value as the price of the underlying asset increases.

2.6 Commodities Markets


Commodities are different in that they are the oldest trading instruments and need to
be physically delivered, with custom settlement terms. Participants need to pay storage
and maintenance costs. Their forward prices are determined by the laws of demand and
supply and these are closely correlated with environmental factors.
Most commodities markets are exchange-screen based. OTC markets in commodities
is generally larger than the corresponding exchange market. Instruments traded on
commodities markets include the following:
- Energy: Crude Oil, Natural Gas, Coal; Refined Products: Fuel Oil, Jet Fuel, Gasoline,
Diesel Fuel, Naphtha
- Base Metals: Copper, Aluminium, Lead, Nickel, Tin, Zinc
- Precious Metals: Gold, Silver
- Agricultural Products: Corn, Sugar, Coffee, Wheat, Cotton
18

Open University of Mauritius - Financial Theory & Practice

2.7 Real Estate Markets


Real estate is usually considered to be buildings and buildable land, including offices,
industrial warehouses and retail space. Real estate is a form of tangible assets, one that
can be touched and seen, as opposed to financial claims that are recorded as pieces of
paper. Real estate is an important investment category. In many countries, domestic real
estate is a common investment vehicle for pension funds and life insurance companies.
In various countries, pooled funds have been created with the specific purpose of real
estate investment.

2.8 SUMMARY
In this unit, you explored various types of markets and products while appreciating
the financial products on offer on the Stock Exchange of Mauritius and its scope for
development. The attention is now turned onto one of the most important concept in
finance: The Time Value of Money. Going forward in time, the present value and future
value of money is not the same. Unit Three offers further clarification.

2.9 Tutorials
Question 1

With the help of an example, discuss how stocks are issued on the market

Question 2

Differentiate between futures, forwards, swaps and options

Question 3

Money markets and bond markets are believed to be the same and are used
interchangeably. Discuss.

Open University of Mauritius - Financial Theory & Practice

19

20

Open University of Mauritius - Financial Theory & Practice

UNIT

THE TIME VALUE


OF MONEY

Unit Structure
3.0 Overview
3.1 Learning Objectives
3.2 Time Value of Money
3.3 Factors Influencing the Time Value of Money
3.4 Simple and Compound Interest
3.5 Present Value of Future Cash Flows
3.6 Introduction - Annuity
3.7 Future Value of an Annuity
3.8 Summary
3.9 Tutorial
3.10 Suggested Readings

Open University of Mauritius - Financial Theory & Practice

21

3.0 OVERVIEW
This unit focuses on the time value of money. This chapter is of particular importance in
that cash that is received in the future has a different value from the cash that is received
presently. Future cash flows are subject to inflation and risk across time and these
variables need to be factored in the value of any future cash flows arising. Additionally,
this unit will introduce the concept of simple and compound interest.

3.1 LEARNING OBJECTIVES


By the end of this Unit, you should be able to do the following:
1. Understand the rationale behind the time value of money and the need to take into
consideration the concept of discounting future cash flows and bring them back
to the present value.
2. Work out the differences between simple and compound interest theory and
perform calculations thereon.
3. Calculate the present value and future value of annuities arising.

3.2 TIME VALUE OF MONEY


We start with the case of an individual who is offered a choice: Receive Rs 1,000 now or
in a years time. It is assumed that there is no inflation and no risk affecting the receipt
of the money in a years time.
A rational investor will normally elect to receive the money now. To the investor,
this choice implies that Rs 1,000 that is received now is more valuable received one
year later.
Hence, the value of the cash flow depends on when the cash flow is to occur. The further
the timing of the cash flow, the lower will be its value now. The essence of this statement
is that going forward, we expect the inflation rate and further risks to affect the receipt
of any cash inflow. Since the inflation rate and risks can only at best be forecasted, the
cash inflow will be discounted in such a way that it has a lower value.
An individual will normally want to use his money for present consumption or save
for future consumption. If he foregoes his current consumption and saves the money
for latter use i.e. he is willing to receive the Rs 1,000 in one years time, he must be
given some sort of compensation and this is called interest or can also be referred to
as his opportunity cost of capital. In effect, Interest is the price for foregoing present
consumption to increase future consumption.
Time value of money: simple illustration
Suppose an investor requires Rs180 in 1 years time and decides to invest Rs100 now.
The investor thus requires compensation of 80 cents for every Rs1 of consumption
he foregoes today. The investors time value of money or required rate of return is
therefore:
FV
180
-1 =
-1 = 0.080 or 80%
PV
100
Hence, the investors return over the period of 1 year is equal to 80%.

22

Open University of Mauritius - Financial Theory & Practice

3.3 FACTORS INFLUENCING


THE TIME VALUE OF MONEY
(a) The real risk-free rate of return
Given that there is no future inflation and no uncertainty in the receipt of the future cash
flows, an investor will require some return for giving up his or her present consumption
in order to increase future consumption. This return is the real risk free rate of return.
Such a return is attainable by investing in Government issued financial instruments such
as Government Treasury Bonds which are assumed to be risk-free, i.e. there is no risk
of default from the Government.

(b) Inflation
If inflation and hence prices do increase, the purchasing power of money declines over
time. Investors will therefore demand compensation in terms of an inflation premium.
Real risk-free rate + inflation premium = Nominal risk free return

(c) Risk
If there are uncertainties over the promised future cash flows, the investor will require a
risk premium. The level of risk and uncertainty (on which the investment is subject to)
will influence the amount of the risk premium.
Nominal risk adjusted return = Nominal risk-free return + risk premium.

3.4 SIMPLE AND COMPOUND INTEREST


Simple interest
Note: interest earned is fixed to the principal amount invested (i.e. the initial amount
invested) only
Suppose you want to invest Rs20,000 for 2 years @ 13% simple interest per annum.
Future value (FV)


= 20,000 + (20,000 x 0.13) + (20,000 x 0.13)


= 20,000 [1 + (1 x 0.13) + 1 x 0.13) ]
= 20,000 [1 + (0.13 x 2) ]
= 25,200

Generalizing on the above results yields the following formula:


FV = PV [1 + r x n ]
Where:
FV =
PV =
r =
n=

Future value
Present value
Proportional rate of interest per period
Number of periods

Compound interest

Note: Interest earned is not fixed to the principal amount invested (i.e. the initial amount
invested) only; subsequent interest payments depend on previous interest payments as well.
Open University of Mauritius - Financial Theory & Practice

23

Illustration 1:
Suppose you want to invest Rs20,000 for 2 years @ 13% compound interest
per annum.
FV



= 20,000 + (20,000 x 0.13) + [20,000 + (20,000 x 0.13)] x 0.13


= 20,000 (1 + 0.13) + 20,000 (1 + 0.13) x 0.13
= 20,000 (1 + 0.13) [1 + 0.13]
= 20,000 (1 + 0.13)2
= 25,538

Generalising from the above:


FV = PV (1 + r)n

Illustration 2:
You borrow $4,000 from the MCB to further invest in your business. The contract
terms states that you need to pay 10% interest compounded annually. How much
will you owe in 5 years if you return the banks $4,000 plus interest?

FV = PV (1 + r)n = 4,000 (1.10)5 = $6,442.04

Illustration 3:
How much money must be invested today earning an 11% interest rate compounded
annually to have $500,000 in 5 years?
Note: You know that you want to earn $500,000 in 5 years time. Therefore, future value
= $500,000 and n = 5 years since interest is being compounded annually (alternatively,
it could have been said that the money is being compounded monthly, in which case n
would have equaled to [5x12 = 60] and the interest rate would have been [0.11/12]%)


PV = FV [1/(1+r)]t = 500,000 [1/(1.11)5] = $296,725.7

Illustration 4:
Shares in a sugar plant sell for $4,000 today and will be worth $5,500 in 6 years.
What is the rate of return expressed as an annually compounded interest rate?

r = (FV/PV)(1/n) - 1

= (5,500/4,000)1/6 - 1

= 5.45%

Discrete Compounding
In this section, we examine investments paying interest more than once a year. For
instance, many banks offer a monthly interest rate that compounds 12 times a year. In
such an arrangement, they pay interest on interest every month. Financial institutions
often quote an annual interest rate that we refer to as the stated annual interest rate or
quoted interest rate and is denoted by rs.

24

Open University of Mauritius - Financial Theory & Practice

With more than one compounding period per year, the future value formula can be
expressed as:

FVN = PV [ 1+ (rs/m)mN
Where:
rs the stated annual interest rate
m the number of compounding periods per year
N stands for the number of years
Illustration 5
Suppose your bank offers you a certificate of deposit with a two-year maturity and
a stated annual interest rate of 8 percent compounded quarterly. You decide to
invest $10,000. What will the Certificate of Deposit be worth at maturity?
PV = 10,000
rs = 8%
N = 2 years
m=4
FVN = PV [ 1+ (rs/m)mN
FV2 = 10000 [ 1+ (8%/4)4x2 = $11,716.59

Continuous Compounding
The predecing discussion on compounding periods illustrates discrete compounding,
which credits interest after a discrete amount of time has elapsed. If the number of
compounding periods per year becomes infinite, the interest is said to compound
continuously. The expression for the future value of a sum in N years with continuous
compounding is:

FVN = PVersN
The termersN is the transcendental number e = 2.7182818 raised to the power rsN.
Illustration 6
Suppose a $10,000 investment will earn 8 percent compounded continuously for
two years. Calculate the future value of the investment.
PV = $10,000
rs = 8%
N=2
FVN = PVersN
FVN = 10000e0.08(2) = $11,735.11
With the same interest rate but using continuous compounding, the $10,000 investment
will grow to $11,735.11 in two years, compared with $11,716.59 using quarterly
compounding, as shown in the previous example.

Open University of Mauritius - Financial Theory & Practice

25

3.5 PRESENT VALUE OF FUTURE CASH FLOWS


The future value of an investment of Rs1,000 @ 10% compounded annually will be:
After 1 year: 1,000 x 1.10 = Rs1,100
After 2 years: 1,000 x 1.102 = Rs1,210
We have simply applied the following formula:

FV = PV (1 + r )n
Now, lets turn the question around! Given that interest is 10% compounded annually
how much we need to invest now to obtain:
Rs1,100 after one year?
Simply make PV become subject of the above formula
PV =

FV
1,100
=
= 1,000
n
(1+r)
(1+0.10)1

Rs1,210 after 2 years


PV =

FV
1,100
=
= 1,000
n
(1+r)
(1+0.10)2


The future cash flows have been discounted. Discounting is therefore a process, which
inverts the compounding process to provide the present value of the future cash flows.

Example:

Given that an investment will produce Rs1,050 at the end of year 1, Rs1,102.50
at the end of year 2 and Rs1,157.63 at the end of year 3, Calculate the present
value of the investment if the investor requires a rate of return of 5% compounded
annually.
PV =

1,050
1,102.50
1,157.63 = 1,000
+
+
1,051
1,052
1,053


By discounting each of the future cash flows occurring at the end of year 1, 2 and 3,
what we are doing is bringing back all future cash flows to the present value so that they
may be compared. Note that the discounting factor in year three (1/1.053 = 0.864) is
greater than the discounting factor in year one (1/1.05 = 0.952). This shows that future
cash flows that are more distant in the future have a greater discounting factor since they
are more uncertain than those arising at the present value or the first few years.
Therefore, if the present worth of this investment taking into consideration the investors
time value of money (5%) is Rs3,000. In effect, Rs3,000 represents the maximum sum
the investor is prepared to pay now for undertaking the investment. The return on the
investment will be exactly 5% if the investor pays Rs3,000 for the investment; if the
investor pays less than Rs3,000, he will be earning a return of more than 5%. However,
if the investor pays more that Rs3,000, a return of less than 5% will be earned.
In conclusion, we can say that the worth of any asset is currently the present value of the
assets future income stream.

26

Open University of Mauritius - Financial Theory & Practice

3.6 Introduction - ANNUITY


An annuity is a succession of predetermined identical payments or receipts made
over consistent time intervals. Examples of annuities are weekly rent, monthly wages,
monthly insurance premiums, and monthly loan repayments amongst others.
In an ordinary annuity, the set of equivalent payments/receipts take place at the last
part of the time interval e.g. rent paid at the end of each month.
In a due annuity, the set of equivalent payments/receipts take place at the beginning of
the time interval e.g. rent paid at the beginning of each month.
A perpetual annuity, whether ordinary or due, is one which carries on forever.

Present value of an ordinary annuity A

Let P denote present value, A being the annuity value and let n denote the number
of terms
P=

A
A
A
A
+
+ .................. +
+
(1+r)1 (1+r)2
(1+r)3
(1+r)n

Multiply [1] by

[1]

1 gives:
(1+r)

P
A
A
A
A
A
=
+
+ .................. +
+
+
(1+r)1
(1+r)2 (1+r)3
(1+r)4
(1+r)n
(1+r)n+1

[2]

Taking [1] - [2] gives:


P-

P
A
=
(1+r)1 (1+r)1

A
(1+r)n+1

[3]

Multiplying [3] by (1+r) gives:


P (1+r)-P = A -

A
(1+r)n

P + Pr - P = A -

A
(1+r)n

Pr = A

P=A

1-

A
(1+r)n

1-

A
(1+r)n
r

[4]

Open University of Mauritius - Financial Theory & Practice

27

Present value of an ordinary annuity A to infinity


From equation [4] above as n tends to infinity

P=

A
r

[5]

Present value of a due annuity A


P=A+

A
A
A
A
+
+ .................. +
+
(1+r)1 (1+r)2
(1+r)3
(1+r)n-1
1-

P=A+A

[6]

A
(1+r)n-1
[7]

Illustration 1:

Calculate the present value of an ordinary annuity of Rs20,000 per year for five
years if the interest rate is 12% per year.
1P = 20,000 x

1
(1+0.12)5

= Rs 72,096

0.12

Illustration 2:

Calculate the present value of a due annuity of Rs20,000 per year for five years if
the interest rate is 12% per year.

1P = 20,000 + 20,000 x

1
(1+0.12)5-1
0.12

= Rs 80,747

3.7 FUTURE VALUE OF AN ANNUITY


Future value of an ordinary annuity

The future value (F) of an annuity invested each period (at interest per period r) beginning
one period from now for n periods is:
F = A + A (1+r)1 + A (1+r)2 + A (1+r)3 + ........................ A (1+r)n-2 + A (1+r)n-1
2nd Annuity

Last annuity

[1]

1St Annuity

Multiply throughout by (1 + r )
F (1+ r) = A (1+r)1 + A (1+r)2 + A (1+r)3 + ................... + A (1+r)n-1 + A (1+r)n

28

Open University of Mauritius - Financial Theory & Practice

[2]

Take 2 1
F (1+ r) F = A (1+r)n A
F + Fr F = A (1+r)n A
F=A

(1+r)n - 1
r

Illustration 1

What is the future value of a 5 year ordinary annuity, if the annual interest is 10%, and
the annual payment is $3,000?
F= 3000{(1+0.1)5-1/0.1} = $ 18,315.3

Illustration 2

Assume that an individual plans to retire at the age of 50, with the life expectancy of
30 years. He expects to spend Rs 50,000 at the end of each year during your retirement.
How much money does he need to save by the age of 50 (lump sum) to support his post
retirement consumption expenditure? Assume an interest rate 7%.
Lump sum at P50 = 50,000 + 50,000 + 50,000
+ ......... + 50,000 =

(1 + 0.07) (1 + 0.07)2 (1 + 0.07)3
(1 + 0.07)30

1= 50,000

1
(1.07)30
0.07

= Rs 620,452.1

Future value of a due annuity

The future value (F) of an annuity invested each period (at interest per period r) starting
now for n periods is:
F = A (1+ r) = A (1+r)2 + A (1+r)3 + ................... + A (1+r)n

[1]

Multiply throughout by (1 + r )
F(1+r) = A (1+r)2 + A (1+r)3 + A (1+r)4 +................... + A (1+r)n + A (1+r)n+1 [2]
Term before last term

Open University of Mauritius - Financial Theory & Practice

29

Take 2 1
F (1+ r) F = A (1+r)n+1 A (1+r)
F + Fr F = A (1+r)n+1 A (1+r)

F = A(1+ r)

(1+r)n - 1
r

Illustration 1

What is the future value of a 5 year due annuity, if the annual interest is 10%,
and the annual payment is $3,000?
F= 3000 (1+0.1) {(1+0.1)5-1/0.1} = $ 20,146.83

3.8 SUMMARY
For economic progress to be possible, there must be a universally applicable time value
of money, even in a risk-free environment. This fundamental concept introduced in this
chapter gives rise to the techniques of capitalization, discounting and net present value,
described in Unit 4.

3.9 Tutorials
Question 1

On 1 January 2012, Rs400, 000 was borrowed by Mr Nuri from a bank for a period of
10 years at a fixed annual rate of interest of 15%. Mr Nuri will reimburse interest and
principal by equal annual payments. Calculate the annual payment if:
The first annual payment is effected on 31 December 2012.
The first annual payment is effected on 1 January 2012.

Question 2

Compute the size of the fund at the end of the period if a company decides to set up a
fund for its employees with an initial payment of Rs30,000 compounded six-monthly
over a five year period at a six-monthly interest of 5%. Also, calculate the effective
annual interest rate.

Question 3

On January 1, 2013, you will deposit $2,000 into a savings account that carries a 10%
interest per annum.

(i) If the bank compounds interest annually, how much will you have in your
account on Jan 1,2017?


30

(ii) What would your Jan 1,2017 balance be if the bank compounds interest on a
semi annual basis?
(iii) Calculate the effective annual rate of interest if the bank compounds interest on
a quarterly basis.

Open University of Mauritius - Financial Theory & Practice

3.10 Suggested Readings


Fabozzi, F.J., Modigliani, F., Jones, F.J., and Ferri, M.J., Foundations of Financial
Markets and Institutions. Third or Latest Edition
Zvi Bodie, Alan Marcus and Alex Kane Investments, 6th or Latest Edition by
McGraw-Hill Higher Ed.
Wilmott, P., Quantitative Finance, Vol 1 & 2, John Wiley & Sons, Reprinted, 2003.

Open University of Mauritius - Financial Theory & Practice

31

32

Open University of Mauritius - Financial Theory & Practice

UNIT

CAPITAL INVESTMENT
APPRAISAL

Unit Structure
4.0 Overview
4.1 Learning Objectives
4.2 Introduction
4.3 Identifying the Projects Cash Flows
4.4 What Does it Mean to Discount a Sum?
4.5 Methods of Investment Appraisal
4.5.1 The Net Present Value (NPV)
4.5.2 The Internal Rate of Return (IRR)
4.5.3 The Payback Period
4.5.4 The Accounting Rate of Return (ARR)
4.6 Summary
4.7 Tutorial
4.8 Suggested Readings

Open University of Mauritius - Financial Theory & Practice

33

4.0 OVERVIEW
Capital Budgeting is the process that companies use for decision making on capital
projects those projects with a life of a year or more. This is a fundamental area of
knowledge for financial analysts for many reasons. Capital Budgeting undergirds the
most critical investments for many corporations their investment in long term assets.
The process of evaluating long-term investment decision is referred to as Investment
Appraisal. The decision whether or not to select a project will usually depend on the
stream of cash flows which are generated over a given time period. These cash flows
will undergo rigorous transformations under different investment appraisal techniques,
namely: the Net Present Value (NPV) method, the Internal Rate of Return (IRR) method,
the Payback method and Accounting Rate of Return (ARR).

4.1 LEARNING OBJECTIVES


By the end of this Unit, you should be able to do the following:
1. Explain the importance of the capital budgeting process
2. Assess the issues in relation to the identification of a projects relevant cash flows.
3. Assess the difference between discounting (net present value and internal rate
of return) and non-discounting cash flow techniques (payback and accounting rate
of return).
4. Assess the feasibility of a particular project using different investment appraisal
techniques
5. Examine the comparative advantages and disadvantages of the investment appraisal
techniques.

4.2 INTRODUCTION
The specific capital budgeting procedures that a manager uses depend on the managers
level in the organization, the size and complexity of the project being evaluated, and the
size of the organization.
The typical steps in the capital budgeting process are as follows:
Step One:
Generating Ideas - Investment ideas can come from anywhere, from the top or the
bottom of the organization, from any department or functional area, or from outside the
company. Generating good investment ideas to consider is the most important step in
the process.
Step Two:
Analysing Individual Proposals This step involves gathering the information to
forecast cash flows for each project and then evaluating the projects profitability.
Step Three:
Planning the capital budget The company must organize the profitable proposals
into a coordinated whole that fits within the companys overall strategies, and it must
also consider the projects timing. Some projects that look good when considered in
isolation may be undesirable strategically. Because of financial and real resource issues,
the scheduling and prioritizing of projects is important.
Step Four:
Monitoring and Post-Auditing In a post-audit, actual results are compared to planned
or predicted results, and any differences must be explained. For example, how do
the revenues, expenses, and cash flows realized from an investment compare to the
predictions?

34

Open University of Mauritius - Financial Theory & Practice

Post-auditing capital projects is important for several reasons.


First, it helps to monitor the forecasts and analysis that underlie the capital budgeting
process. Systematic errors, such as overly optimistic forecasts, become apparent.
Second, it helps improve business operations. If sales or costs are out of line, it will
focus attention on bringing performance closer to expectations if at all possible.
Finally, monitoring and post-auditing recent capital investments will produce concrete
ideas for future investments. Managers can decide to invest more heavily in profitable
areas and scale down or cancel investments in areas that are disappointing.
Capital budgeting is a cost-benefit exercise. At the margin, the benefits from the
improved decision making should exceed the costs of the capital budgeting efforts. The
aim of the capital budgeting process is thus to identify projects that support the goal of
maximizing the market value of the firm.

4.3 IDENTIFYING THE PROJECTS CASH FLOWS


One of the key priorities in the capital budgeting process is to identify the relevant cash
flows accruing to the project under consideration. These relevant cash flows should be
satisfying the following criteria.
Accounting profits are irrelevant and only cash flows ought to be considered
Profit is an accounting concept to report performance of a firm by accountants in each
accounting year. Basically, accounting profit takes both cash sales and credit sales in
its computation of sales. However, credit sales are not cash flows as they have not yet
been converted into cash. Hence, we disregard all non-cash flows for the purpose of
investment appraisal.
Incremental cash flows are the relevant cash flows
All sunk costs, committed future costs, non-incremental fixed costs and overheads
should not be taken into account when evaluating the feasibility of the project. Only
incremental cash flows are relevant (i.e. those cash flows that will change as a direct
result of undertaking the project).
All financing cash flows should be disregarded
The present value of the financing cash flows is represented by the projects initial outlay.
To avoid double counting, we must not include interest and principal repayments over
and above the initial capital outlay.

A company wishes to replace a machine which will cost Rs150, 000 and has a
three year life. The company decides to finance the purchase of this machine by
undertaking a loan from the bank. The bank charges an interest rate of 12% per
annum with the principal being paid at the end of the third year.
Year
Initial Outlay
Interest

(18,000)

(18,000)

(18,000)

(150,000)

Principal
Repayment

(150,000)

The above representation is wrong since the present value of the loans cash flows is the
machine initial outlay. The interest and principal are financing cash flows and should
be ignored.
Open University of Mauritius - Financial Theory & Practice

35

In fact, the following calculation shows:


PV of loan cash flows:

18,000 18,000
18,000 + 18,000
+
= Rs 150,000
1
2 +
1.12
1.12
1.123

 orking capital adjustment needs to be incorporated in the calculation of


W
relevant cash flows

Changes in the level of working capital represent changes in cash flows. Basically, cash
outflows are represented by increases in the working capital requirement such as increase
in debtors or stocks. In fact, the increase in debtors represents an investment in working
capital as payment of sales is not made immediately. Also, increases in stock are due to
lower stock turnover. On the other hand, cash inflows are represented by decreases in
the working capital requirement. For instance, reduction in stocks may be due to greater
stock turnover or reduction in debtors due to earlier settlement of debt by debtors

 ake into account all opportunity costs that may arise from undertaking
T
the project

An opportunity cost is what resource is worth in its next-best use. For example, if a
company uses some idle property, what should it record as the investment outlay: the
purchase price several years ago, the current market price or nothing? If you replace an
old machine with a new one, what is the opportunity cost? If you invest Rs 10million,
what is the opportunity cost? The answers to these questions are respectively: the current
market value, the cash flows the old machine would generate, and Rs 10million (which
you could invest elsewhere)

Taxation

In the appraisal of an investment project, focus is on the cash flows that will be generated
by the project and that which are available for shareholders. As such, we want to assess
the after tax cash flows of the project. It is important to note that the taxation charge is
levied on the taxable profits of the business and not on the net cash flows.

4.4 What does it mean to discount a sum?


To discount means to calculate the present value of a future cash flow.
Discounting into todays rupees helps us to compare a sum that will not be produced
until later. Technically speaking, to discount is to depreciate the future. It is to be
more rigorous with future cash flows than present cash flows, because future cash
flows cannot be spent or invested immediately.
First, take tomorrows cash flow and then apply to it a multiplier coefficient below 1,
which is called a discounting factor. The discounting factor is used to express a future
value as a present value, thus reflecting the depreciation brought on by time.
Consider an offer whereby someone will you Rs1,000 in 5 years. As you will not receive
this sum for another 5 years, you can apply a discounting factor to it, for example, 0.6.
The present, or todays value of this future sum is then 600. Having discounted the
future value to a present value, we can then compare it to other values. For example,
it is preferable to receive 650 today than 1000 in 5 years, as the present value of 1000
5 years from now is 600, and that is below 650.

36

Open University of Mauritius - Financial Theory & Practice

Discounting make is possible to compare sums received or paid out at different dates.
Discounting is based on the time value of money. After all, time is money. Any sum
received later is worth less than the same sum received today. Remember that investors
discount they demand a certain rate of return. If a stock pays you Rs110 in one year and
you wish to see a return of 10% on your investment, the most you would pay today for
the security (i.e. the present value) is 100.
Discounting is calculated with the required return of the investor. If the investment
does not meet or exceed the investors expectations, he will forego it and seek a better
opportunity elsewhere.

4.5 METHODS OF INVESTMENT APPRAISAL


The investment appraisal techniques can be classified into two main categories,
discounting and non-discounting techniques.

Discounted Cash Flow Techniques: Takes into account the time value of money
Net present value
Internal Rate of Return.
Discounted payback

Non-Discounting Techniques : Ignores the time value of money


Payback period
Accounting rate of return

We will now explore these four investment appraisal techniques with the help of the
following example:
Glass Company is a manufacturer of plates for the hospitality sector. Currently,
the firm is considering whether to invest in one of the two proposed machines,
Machine A and Machine B. The Management of Glass Company wishes to revamp
its existing operations and firmly believes that such securing such a machine will
automate its processes and contribute to further savings on its production line.
The following information is provided:
Investment Case: Glass Company
Machine A
(MUR)
Investment outlay (payable immediately)

Machine B
(MUR)

250,000

325,000

Year 1

80,000

90,000

Year 2

90,000

91,000

Year 3

100,000

92,000

Year 4

1100,000

93,000

Annual cost saving:

The rate of return that is expected on projects with similar risk is 7%.
Note: The annual cost savings imply that the company will receive these cash flows and as
such are relevant cash flows for the company

Which one of the above machines should be purchased, should the company decide to
move forward with its investment decision?

Open University of Mauritius - Financial Theory & Practice

37

4.5.1 THE NET PRESENT VALUE (NPV)

Calculating the NPV of a project is conceptually easy. There are basically two steps
to be followed:
1. Write down the net cash flows that the investment will generate over its life
2. Discount these cash flows at an interest rate that reflects the degree of risk inherent
in the project
The resulting sum of discounted cash flows equals the projects net present value.
The NPV Decision Rule says to invest in projects when the net present value is positive
(greater than zero):
NPV > 0 Invest

NPV < 0
Do Not Invest
The NPV Rule implies that firms should invest when the present value of future cash
inflow exceeds the initial cost of the project. The firms primary goal is to maximize
shareholder wealth. The discount rate r represents the highest rate of return (opportunity
cost) that investors could obtain in the marketplace in an investment with equal risk.
When the NPV of cash flow equals zero, the rate of return provided by the investment is
exactly equal to investors required return. Therefore, when a firm finds a project with a
positive NPV, that project will offer a return exceeding investors expectations
NPV of Machine A project
250,000 +

80,000 90,000 100,000


110,000
+
+
= Rs 68,924.10
+
1.07
1.072
1.073
1.074

NPV of Machine B project


325,000 +

90,000 91,000
92,000
93,000
+
+
= Rs 15,356.27
+
1.07
1.072
1.073
1.074

What does an NPV of Rs 68,924.10 for Machine A project stands for?


NPV shows how much more or how much less in money terms the project is earning in
comparison with an alternative project that have similar risk levels. A positive NPV of
Rs 68,924.10 indicates that the shareholders wealth will increase by this amount should
they decide to go forward with the project. Hence, this implies that the project has a
higher return than on other similar risk level investments.
Basically, if Rs 250,000 were invested on a similar risk level project, a return of 7% will
be obtained. However, it is noted that the NPV of Machine B project is negative, which
shows that this project is earning a return less than 7%. This implies that the company
should not invest in that project.
Advantages of NPV:
Takes into account the risk levels of the projects- Basically, the risk is reflected in
the discount rate as it is the return that is obtained on projects having similar risks;
Decision to reject or accept a project is a relative one not an absolute one- in fact, it
is relative to what the foregone alternative will earn;
Considers the time value of money concept;
Take into consideration all cash flows arising;
It is an absolute measure of return;
Leads to maximization of shareholder wealth

38

Open University of Mauritius - Financial Theory & Practice

Disadvantages of NPV
It is difficult to explain to managers. To understand the meaning of the NPV
calculated requires an understanding of discounting. The method is not intuitive as
techniques such as payback;
It requires the knowledge of the cost of capital;
It is relatively complex.

4.5.2 THE INTERNAL RATE OF RETURN (IRR)


If net present value (NPV) is inversely proportional to the discounting rate, then there
must exist a discounting rate that makes NPV equal to zero. The discounting rate that
makes NPV equal to zero is called the internal rate of return or IRR. The IRR is the
actual rate of return of the project.
The decision making rule is very simple: if an investments internal rate of return is
higher than the investors required return, he will make the investment. Otherwise, he
will abandon the investment.
The IRR of Machine A project is calculated as follows:
80,000
90,000
100,000
110,000
+
+
= Rs 68,924.10
+
(1+IRR) (1+IRR)2
(1+IRR)3
1.074

250,000 +

A variety of computer software can be used to solve the above equation. Alternatively,
linear interpolation, a mathematical technique can be used to estimate the IRR. This
consists of determining two discount rates such that one of them produces a positive
NPV and the other produces a negative NPV.
For Machine As project, a discount rate of 15% produces a positive NPV of Rs
16,262.63 whilst a discount rate of 20% produces a negative NPV of Rs 9,915.12. By
linear interpolation, we apply the following formula to estimate IRR:
IRR= LDR +

=15% +

HDR - LDR
x NPVLDR
NPVLDR - NPVHDR

20 - 15
x 16,262.63 = 18%
16,262.63 + 9,915.12

Where:

HDR = Higher discount rate

LDR = Lower discount rate

NPVLDR = NPV corresponding to the LDR


NPVHDR = NPV corresponding to the HDR
The IRR for the Machine B project is approximately equal to 4.90%. (Calculate it!)

Open University of Mauritius - Financial Theory & Practice

39

IRRs and NPVs

The IRR only compares the project yield with that of the capital market. In other words,
it answers the question can the project produce a higher return than the capital market.
However, it does not give an indication to judge between alternative projects like the
NPV i.e how much more or less the project is earning relative to the risk equivalent
capital investment alternative.
Another problem arising with the IRR is that this method gives rise to multiple
IRRs when uneven cash flows are considered. Out of these two investment appraisal
techniques, NPV always prevail over IRR.
Advantages of IRR
Considers the time value of money
Is a percentage and is readily understood
Uses cash flows and not profits
Consider the whole life of the project
Means a firm selecting projects where the IRR exceeds the cost of capital, should
increase shareholders wealth
Disadvantages
It is not a measure of absolute profitability
Interpolation only provides an estimate and an accurate estimate requires the use
of spreadsheet program
It is fairly complicated to calculate
Non-conventional cash flows may give rise to multiple IRRs which means the
interpolation method cannot be used.

4.5.3 THE PAYBACK PERIOD

The payback period measures the number of years it takes to recover the initial cash
outlay on a real asset. A company can also define a cut-off date i.e. set the number of
years over which it wants to recuperate the initial amount invested. E.g. If a certain
company sets its cut-off date to 2 years, it implies that the company wants to recuperate
its investment within two years. In the event that a project offers a payback period of
3 years, the company will refuse the investment since the payback period (3 years) is
greater than the cut-off date of 2 years. In practice, the faster a firm recoups the initial
capital outlay, the less risky are the projects.
The advantages of the payback period method include the following:
Easy to understand and simple to compute
When capital is scarce or in short supply, it could be argued that projects that
returned the expenditure rapidly are the best ones.
It is useful in certain situations (rapidly changing technology and improving
investment conditions)
It favours quick return, which in turn help company growth, minimizes risk and
maximizes liquidity
It uses cash flows, not accounting profits
The drawbacks of this method are:
Ignore the time value of money.
It does not assess the impact of cash inflows arising after the payback period, which
could greatly affect the projects profitability.
It is subjective no definitive investment signal
It ignores project profitability
40

Open University of Mauritius - Financial Theory & Practice

Consider the following table where the payback period for Machine A is considered:

Year

Cash Flow

Cumulative Cash
Flow

Payback Period

(250,000)

(250,000)

80,000

(170,000)

90,000

(80,000)

100,000

20,000

(80,000/100,000) x
(1 year or 12 months)
= 0.8 years or 9.6 months

110,000

Payback period for Machine project A is 2.8 years or 2 years and 10 months.
Payback period for Machine project B is 3.6 years or 3 years and 7 months. (Calculate it!)
Machine A project has the shortest payback period as it recoups initial capital expenditure
earlier that the Machine B project. Hence, management will consider going forward
with the Machine A project under the payback period method.
The Discounted Payback Period method
The Discounted payback period method has been introduced to cater for the time value
disadvantage borne by the payback period. By discounting the future cash flows arising
from the project, you will have taken into consideration the time value of money.
Consider the following example:

Year

Cash Flow

Discounted
Cash Flow
(at 7%)

(250,000)

(250,000)

(250,000)

80,000

74,766

(175,234)

90,000

78,609

(96,625)

100,000

81,629

(14,996)

68,922

14,996/83,918
x 12 Months
= 3 years and 3 Months

110,000

83,918

Cumulative
Cash Flow

Payback Period

Hence, using the discounted payback period, as opposed to the normal payback period,
the payback period has increased from 2 years and 10 months to 3 years and 3 months.
This new method has taken into consideration the riskiness of the cash flows and has
accounted for the risk by applying the time value of money to it. This would reflect a
more conservative approach by management as regards to the payback period method.

Open University of Mauritius - Financial Theory & Practice

41

4.5.4 THE ACCOUNTING RATE OF RETURN (ARR)


The ARR measure the projects profitability over the entire asset life. It compares the
average accounting profit of the project with the initial or average amount of capital
invested. The ARR can be calculated as follows:
ARR =

Average annual profit


x 100
Initial (or average) capital invested

We assume that straight line depreciation method is used in each case.


The average accounting profit and the ARR (based on initial capital invested) for
Machine A and Machine B projects are as follows:
Machine Project A
Cash flow
Depreciation
Profit

Year 1
Rs

Year 2
Rs

Year 3
Rs

Year 4
Rs

80,000

90,000

100,000

110,000

(62,500)

(62,500)

(62,500)

(62,500)

17,500

27,500

37,500

47,500

The average profit over the years is: (17,500 + 27,500 + 37,500 + 47,500) / 4 = Rs 32,500
Hence, the accounting rate of return for Machine project A is given by:
(32,500/250,000) x 100 = 13%
Calculate the accounting rate of return for Machine Project B.
The advantages of this method are:
easy to understand and simple to compute
It takes into account the importance of profitability.
The drawback of this method:
It ignore the time value of money
It makes use of accounting data instead of cash flow data

4.6 SUMMARY
Unit 4 presented various techniques for appraising a project, followed by a discussion
of the strengths and weaknesses of each method. Unit 5 will now build further on the
concept of discounting, with a different application. The methodology will now be
applied to the valuation of shares.

42

Open University of Mauritius - Financial Theory & Practice

4.7 Tutorials
Question 1
Rose Company Ltd is considering investment in either project A or project B. The
management of the company has entrusted you the responsibility to determine the
feasibility of the project. Each project has an initial investment of Rs 200m and the
Weighted Average Cost of Capital (or discount rate or opportunity cost of capital) is
22%. Using both discounting and non-discounting investment appraisal techniques,
determine which project should be selected.
Year

Expected Future Cash Flows


Project A

Project B

90M

75M

85M

70M

80M

65M

80M

65M

Question 2
Matt Ltd is about to undertake a new project which entails into investing in a specialised
machine costing Rs155, 000. The project will last for 5 years, after which it is estimated
that the scrap value of the machine will be Rs20,000, to be received at the end of the
sixth year. There is a maintenance cost to ensure that the machine is working properly.
The amount needed at the end of the first year is Rs10,000 and it is estimated that this
amount needs to be increased by 15% at the closing stages of each subsequent year over
the period of the project. Expected revenue through the use of the machine is estimated
at Rs200,000 at the end of the first year, and this will increase by 5% at the end of each
successive year over the period of the project.
(a) Identify the relevant cash flows of the project
(b) Calculate the NPV of the project given that the discount rate of the project is
15 percent.
(c) Estimate the IRR of the project, outlining the disadvantages of using IRR
(d) Calculate the Payback Period of the project assuming a cut-off period of 4 years.

Question 3
A firm is considering two separate capital projects with cash flows as follows:
Year

Project 1

(20,000)

8,000

5,000

5,000

8,000

5,000

Project 2

(30,000)

10,000

10,000

10,000

10,000

11,000

Both projects are of equal risk and the required return on similar risk projects is 10%
(a) 
Based on the NPV criterion, which project will capitalize on the wealth of
shareholders?
(b) Estimate the IRR of each project using linear interpolation.

Open University of Mauritius - Financial Theory & Practice

43

4.8 Suggested Readings


Thomas E. Copeland, J. Fred Weston Financial Theory and Corporate Policy (3rd or
Latest Edition) by Addison Wesley
Brealey and Meyers, Principle of Corporate Finance, McGraw-Hill, 5th or Latest edition

44

Open University of Mauritius - Financial Theory & Practice

UNIT

SHARE VALUATION,
RISK AND RETURN

Unit Structure
5.0 Overview
5.1 Learning Objectives
5.2 Measuring Return on a Share
5.3 Major Categories of Equity Valuation Models
5.4 The Dividend Discount Model
5.4.1 The Constant Growth Dividend Discount Model
5.4.2 Multistage Dividend Discount Model
5.5 Free Cash Flow Valuation Model
5.6 Summary
5.7 Tutorial
5.8 Suggested Reading

Open University of Mauritius - Financial Theory & Practice

45

5.0 OVERVIEW
The equity valuation models used to estimate intrinsic value present value models,
multiplier models and asset-based valuation are widely used and serve an important
purpose. The valuation models presented in this unit are a foundation on which to base
analysis and research but must be applied wisely. Valuation is not simply a numerical
analysis. The choice of model and the derivation of inputs require skill and judgment.
This Unit will focus on the present value models Free Cash Flow valuation and the
dividend valuation models. The key to estimation is to find the intrinsic value of the
share and then compare it to the market price.

5.1 LEARNING OBJECTIVES


By the end of this Unit, you should be able to do the following:
Measure the return on a share
Evaluate whether a share is overvalued or undervalued
Describe the major categories of equity valuation models
Estimate the price of a share under the Dividend Discount Model
Estimate the price of a share under the Free Cash Flow Valuation Model

5.2 MEASURING RETURN ON A SHARE


There are two types of income that shareholders of company can have:
(i) Dividend
(ii) Capital gain/loss
Expected return from holding a share is calculated as follows:
r=

D1 + (p1 - p0)
P0

where D1 is the expected dividend per share


P0 is the current price per share

P1 is the price that is expected next year

Example:
BM stock is selling at Rs 100 a share and investors are expecting the company to
pay a cash dividend of Rs 5 over the next year. Investors also have the information
from their analysts that the price of BM stock will be Rs 120 in one years time.
If they decide to purchase the stock, their expected returns will be as follows.

r=

Rs5 + Rs120 - Rs100


Rs100

= 0.25 or 25%

This expected return is originated from two parts, the dividend yield and the capital gain.

46

Open University of Mauritius - Financial Theory & Practice

Expected return

Expected Dividend Yield + Expected Capital Gain/Loss

D1
D1 - P0
= P +
P0
0
Rs5
= Rs100

Rs120 - Rs100
Rs100

=
0.05 + 0.20
=
5% + 20%
=
0.25 or 25%
However, the actual return for BM stock may turn out to be greater or less than what
investors expect.

5.3 MAJOR Categories of equity


valuation models
Three major categories of equity valuation models are:

(a) Present Value Models

These models estimate the intrinsic value of a security as the present value of the
future benefits expected to be received from the security. In present value models,
benefits are often defined in terms of cash expected to be distributed to shareholders
(Dividend Discount Model) or in terms of cash flows available to be distributed to
shareholders after meeting capital expenditure and working capital needs (Free Cash
Flow Valuation Models).

(b) Multiplier Models

These models are based chiefly on share price multiples or enterprise value multiples.
The former model estimates intrinsic value of a common share from a price multiple
for some fundamental variable, such as revenues, earnings, cash flows, or book value.
Examples of the multiples include price to earnings (P/E, share price divided by earnings
per share) and price to sales (Share price divided by sales per share). The fundamental
variable may be stated on a forward basis (e.g. forecasted EPS for the next year) or
a trailing basis (e.g. EPS for the past year), as long as the usage is consistent across
companies being examined. Price multiples are also used to compare relative values.
The use of the ratio of share price to EPS that is, the P/E multiple to judge relative
value is an example of this approach to equity valuation.

(c) Asset-Based Valuation Models

These models estimate intrinsic value of a common share from the estimated value of the
assets of a corporation minus the estimated value of its liabilities and preferred shares.
The estimated market value of the assets is often determined by making adjustments to
the book value of assets and liabilities. The theory underlying the asset based approach
is that the value of a business is equal to the sum of the value of the businesss assets.

Open University of Mauritius - Financial Theory & Practice

47

5.4 THE DIVIDEND DISCOUNT MODEL


The simplest present value model of equity valuation is the dividend discount model,
which specifies cash flows from an investment in shares to be dividends. If the issuing
company is assumed to be a going concern, the intrinsic value of a share is the present
value of expected future dividends. If a constant required rate of return is also assumed,
then intrinsic value of a share is:
P0 =

D1
D2
D3
D1
+
+ .................. +
+ ..................
+
2
3
(1+r)
(1+r)
(1+r)
(1+r)4

If a companys share offers a perpetual stream of equal dividend payment,


(perpetuity) i.e., D1 = D2 = D3 = = Dt =
Then, P0 =

D1
r

Consider the following example


For the next three years, the annual dividends of a stock are expected to be Rs2.00,
Rs2.10 and Rs2.20. The stock price is expected to be Rs20.00 at the end of three years.
If the required rate of return on the shares is 10%, what is the estimated value of a
share?
The present values of the expected future cash flows can be written as follows:
Value of share =

2
2.10
2.20
20
+
+
+
2
3
(1 + 0.10) (1 + 0.10)
(1 + 0.10)
(1 + 0.10)3

Hence, the value of the share is equal to Rs20.33

5.4.1 The Constant Growth Dividend Discount Model

The assumptions of the Gordon model are:


Dividends are the correct metric to use for valuation purposes
The dividend growth rate is forever
The required rate of return is also constant over time
The dividend growth rate is strictly less than the required rate of return.
If the future dividend of a company is expected to grow at a constant rate, g, then we can
use the following formula to calculate the price of the share:
P0 =

D1
r-g

The above is known as the constant-growth dividend discount model or the Gordon
Growth Model. Due to its assumption of a constant growth rate, the Gordon growth
model is particularly appropriate for valuing the equity of dividend-paying companies
that are relatively insensitive to the business cycle and in a mature growth phase.
Examples might include an electric utility serving a slowly growing area or a producer
of a staple food product.

48

Open University of Mauritius - Financial Theory & Practice

Note: The growth rate of a company can be estimated using earnings retention rate
and the return on equity of the business. Growth rate, g = b x ROE
Where,
b is the earnings retention rate = (1 Dividend payout ratio);
ROE is the return on equity of the company
Example:
Dividend, to be paid in next year, is estimated at D1 = Rs5, the growth rate of dividends
is g = 20%, and the discount rate is r = 25%. Therefore, the share value will be:
P0 =

D1
r-g

P0 =

Rs5
0.25 - 0.20

= Rs100

The constant growth formula can be rearranged as follows to calculate the expected
rate of return:
P=

D1
+g
P0

r dividendyield + growthrate
For BM share, the expected first-year dividend is Rs5 and the growth rate is 20%.
With an initial share price of Rs100, the expected rate of return is
r=

D1
+g
P0

r=

Rs5
+ 0.20
Rs100

r = 0.05 + 0.20
r = 0.25 + 25%

5.4.2 Multistage Dividend Discount Models

Multistage growth models are often used to model rapidly growing companies. The
two stage DDM assumes that at some point the company will begin to pay dividends
that grow at a constant rate, but prior to that time, the company will pay dividends
that are growing at a higher rate than can be sustained in the long run. That is, the
company is assumed to experience an initial, finite period of high growth, prior to the
entry of competitors, followed by an infinite period of sustainable growth. The two
stage DDM thus makes use of two growth rates: a high growth rate for an initial, finite
period followed by a lower, sustainable growth rate into perpetuity.
Consider the following:
The current dividend, D0, is Rs5.00. Growth is expected to be 10 percent a year for
three years and then 5 percent thereafter. The required rate of return is 15 percent.
Estimate the intrinsic value.

Open University of Mauritius - Financial Theory & Practice

49

D1 = 5.00 (1 + 0.10) = 5.50.Growth of dividend in year 1


D2 = 5.00 (1+0.10)2 = 6.05.Growth of dividend in year 2
D3 = 5.00 (1 + 0.10)3 = 6.655.Growth of dividend in year 3
D4 = 5.00 (1 + 0.10)3 (1 + 0.05) = 6.98775.Growth of dividend in year 4, adjusted for
the change in growth in year 4
V3 = (6.98775 / (0.15-0.05)) = 69.8775Thereon, it is assumed that the dividend
will grow at a constant pace. Hence, explaining the use of the Gordon Growth Model
and its subsequent application
V0 =

5.50
6.05
6.655
69.8775
+
= Rs59.68
+
+
2
3
(1 + 0.15) (1 + 0.15)
(1 + 0.15)
(1 + 0.15)4

All these dividends are future expected cash flows and therefore this means that they
are only estimation. Going on this principle, this implies that these future expected cash
flows should be discounted at the required rate of return 15% to account for the time
value of money and returning it back to the present value. This includes value V3 .

5.5 Free Cash Flow Valuation model


Each year, companies produce statements as part of their annual reports. As such no
value of free cash flow is directly available from these statements and therefore, it is
necessary to go back to first principles and start with the profit and loss account, making
adjustments to the profit figure to derive the cash flow.
Free cash flow is calculated as follows:
FCF =
EBIT (Operating income or PBIT)
- Tax on operating income
+ Depreciation and Amortisation
- Capital Expenditure
- Change in Working Capital (An increase in W.C. is deducted and a decrease in
W.C. is added)
Companies, as legal persons have the possibility of unlimited lifetimes. Obviously, when
valuing a company it is impossible to make intelligent forecasts beyond a few years, so
typically, valuation involves two time periods; a forecast period and the time from the
end of that period to infinity. During the forecast period, it is possible to make detailed
assumptions about how changing strategy will affect the components of free cash flow.

Value of company (in case of no growth) =

Value of company (with growth) =

50

FCFt
+
(1 + WACC)t

FCFt
+
(1 + WACC)t

FCFn/WACC
(1 + WACC)n

FCFn/( WACC - g)
(1 + WACC)n

Open University of Mauritius - Financial Theory & Practice

Consider the following:


RsM

2008

2009

2010

2011

2012

2013

2014e

EBIT

185

225

250

271

272

246

235

Tax

-68

-83

-93

-100

-101

-91

-87

+Depreciation &
Amortisation

132

133

138

156

159

162

167

-134

-141

-142

-163

-165

-167

-169

- Change on
working capital

-38

14

-12

-8

-8

-8

-8

Free Cash Flow

77

148

141

156

157

142

138

- Capital
Expenditure

To calculate the value during this period, the FCFs are discounted at the weighted
average cost of capital (WACC). With a WACC of 10%, the calculation is as follows:
Value of Company, V1
= (77/1.1) + (148/1.12) + (156/1.13) + (157/1.14) + (157/1.15) + (142/1.16) + (138/1.17)
= 653
Terminal Value of Company = (Normalised free cash flow / WACC g) / (1+WACC)n
Assuming a growth rate of 5 percent,
Terminal value = (138 x (1+g)) / (0.10 0.05) = 2,898 / 1.106 = 1,635
Hence, total value of the company = 1,635 + 653 = 2,288

5.6 SUMMARY
The aim of this chapter was to present the key parameters used in analysing stocks and
show how the market estimates the intrinsic value of a share. Analysts then base their
investment decisions on that comparison. The next session presents the foundation to
one of the largest and fastest growing segments of global financial markets Fixed
Income Securities. The chapter will introduce some basic features and then build on it
to describe yields and pricing of bonds

Open University of Mauritius - Financial Theory & Practice

51

5.7 Tutorials
Question 1

Total, one of Frances largest corporations and the worlds fifth largest publicly traded
integrated petroleum company operates in more than 130 countries. To meet growing
energy needs on a long-term basis, Total considers sustainability when making decisions.
Selected financial information is available in table 1 below.
Year

2008

2007

2006

2005

2004

DPS
Dividend Per
Share/$

2.28

2.07

1.87

1.62

1.35

Share price/$

39.91

56.83

54.65

52.37

39.66

The analyst estimates the required rate of return to be 19% from CAPM.
Assume that Totals growth rate will mirror the average growth in dividend over the
period considered (D2008 = D2004 x (1 + g)4)
(a) Use the Gordon Growth Model to estimate Totals intrinsic value
(b) Based on the estimated intrinsic, is the share undervalued or overvalued?
(c) What is the intrinsic value of the growth rate estimate is lowered to 13%?
(d) What is the intrinsic value if the growth rate estimate is lowered to 13% and the
required rate of return is increased to 20%

Question 2

A company does not currently pay a dividend but is expected to begin to do so in five
years (at t = 5). The first dividend is expected to be $4.00 and to be received five years
from today. That dividend is expected to grow at 6% into perpetuity. The required return
is 10%. What is the estimated current intrinsic value?

Question 3

The following are the simplified financial statements for J.S. Company, a renowned
retailer in Europe.
Group Income Statement for the Year to 22 March 2008
$
Revenue
17837
16835
Cost of Sales
Gross Profit
1002
Administrative Expenses
502
Other Income
30
Operating Profit
530
Interest Income
83
Interest Expense
132
2
Share in loss from joint ventures
Profit before tax
479
150
Tax
Profit for the year
329

Note: Operating profit is calculated after charging depreciation of 463 and amortisation of 18

52

Open University of Mauritius - Financial Theory & Practice

Group Balance Sheet as at 22 March 2008


Non-Current Assets
2008
Property, Plant and Equipment
7424
Intangible Assets
165
Investment in Joint ventures
148
Other Assets
768
8505
Current Assets
Inventories
681
Trade and Other Receivables
206
Derivative Instruments
4
Cash and Equivalents
719
1610
Total Assets
10115
Current Liabilities
Trade and other payables
Short-term debt
Derivative instruments
Taxes payable
Provisions
Non-Current Liabilities
Other payables
Long Term Debt
Derivative Instruments
Deferred Income Taxes
Provisions and other
Net Assets
Equity
Called up share capital
Share premium account
Capital redemption reserve
Other reserves
Retained Earnings
Total Equity

2007
7176
175
98
212
7661
590
197
1128
1915
9576

2280
118
6
191
10
2605

2267
373
2
65
14
2721

89
2084
18
321
63
2575
4935

33
2090
43
168
172
2506
4349

499
896
680
494
2366
4935

495
857
670
143
2184
4349

(a) Calculate J.Ss free cash flow for 2008. You may assume effective tax rate of 31.3%
(b) Analysts believe that wholesale food prices will rise by 5% each year for the next
three years, but that supermarkets will be able to raise their prices by only 4% a
year during that time. After that, the future is so uncertain that it is best to assume
zero growth in free cash flows. What is the value of J.S. under these circumstances?
You may assume that J.S weighted average cost of capital is 8%.

Open University of Mauritius - Financial Theory & Practice

53

5.8 Suggested Readings


Bodie, Zvi; Kane, Alex; Marcus, Alan J., Investments, Eight Edition
Brealey and Meyers, Principle of Corporate Finance, McGraw-Hill, 5th or Latest edition
Fabozzi, F.J., Modigliani, F., Jones, F.J., and Ferri, M.J., Foundations of Financial
Markets and Institutions. Third or Latest Edition
Thomas E. Copeland, J. Fred Weston Financial Theory and Corporate Policy (3rd or
Latest Edition) by Addison Wesley

54

Open University of Mauritius - Financial Theory & Practice

FIXED INCOME
SECURITIES

Unit Structure
6.0 Overview
6.1 Learning Objectives
6.2 Introduction
6.3 Bond Overview
6.4 Bond Pricing
6.5 Bond Riskiness
6.6 Yield to Maturity
6.7 Practitioners perspective
6.8 Summary
6.9 Tutorial
6.10 Suggested Reading

Open University of Mauritius - Financial Theory & Practice

55

6.0 OVERVIEW
This unit builds up the pedestal for students to steepen their knowledge of how fixed
income securities, more specifically bonds, work and their various functions within the
financial sphere.

6.1 LEARNING OBJECTIVES


By the end of this Unit, you should be able to:
1. Understand the taxonomy related to bonds and how it compares to equity.
2. Price a coupon-bearing bond.
3. Discuss the concept of the yield curve.
4. Understand the drivers of discount rates used for bond pricing.
5. Have an overview of riskiness associated with bond investments.
6. Briefly discuss the rationale behind different practitioners use of debt/bond.

6.2 INTRODUCTION
Broadly speaking, a fixed income security is one which provides an investor with a
stable, known amount of cash flow at known time intervals in exchange of a principal
which will be paid back at the bond expiry date. In the Unit that follows, emphasis will
be laid on such types of instruments only.
However, it should be noted that the definition of fixed income securities is as crisply
defined as above. A non-exhaustive list of other securities that fall within this category
would be swaps, forward agreements and related options but are beyond the scope of
the syllabus. Nevertheless, students should keep in mind that various companies such as
The Mauritius Commercial Bank and Sun Resorts Limited use interest rate and foreign
currency swaps to hedge their exposure to such risks.
Henceforth, the terms fixed income, bond and debt will be used interchangeably.

6.3 BOND OVERVIEW


The Flow of Money

A bond is a promise to make periodic coupon payments and to repay principal at maturity
and is issued by corporations and government units. The terms of the bond contract will
specify the amounts and dates at which payments are to be made. Such payments are
made until the final repayment date which is called the maturity date of the bond. The
time remaining until this final payment is made is known as the time to maturity.
Typically, bonds make 2 types of payments to the bond holder. The specified interest
payments are known as coupons and these are typically made on a semi-annual basis in
countries with mature debt markets, such as the UK and the US. The principal or the
face value is the final payment that is received and is amount that is used to calculate the
coupon payments to be made. They are usually specified in standard increments such
as $1000 bond.
For example, a $1000 with 4% semi-annual coupon payment with maturity in 2 years
will make the following payments to the bond holder. In this case, we assume that the
investor has just received a coupon payment and thus, there is no payment at time 0.

56

Open University of Mauritius - Financial Theory & Practice

Coupons Principal
Time 0: $0 $0
Time 6 months:

$(0.5 * 4% * 1000) = $20

$0

Time 12 months:

$(0.5 * 4% * 1000) = $20

$0

Time 18 months:

$(0.5 * 4% * 1000) = $20

$0

Time 24 months:

$(0.5 * 4% * 1000) = $20

$1000

At the end of the 24 months, both the coupon payment of $20 and the $1000 is received
by the investor.
Diagrammatically, the flow of money is depicted as follows:

The parallels between bonds and shares

1. Bond has a fixed maturity while a share has an infinite life.

2. Bond coupon payments are fixed in most cases while dividend payments are at the
discretion of the firm and vary over time.
3. If a company misses its coupon payments, it is in default but a company can stop
paying out dividends without going into financial distress.
4. As opposed to equity holders, bond holders have no voting rights and ownership
in the firm.
5. Bonds have seniority in bankruptcy i.e. bondholders should be paid first and only
residual cash is for equity holders.
6. Debt interest payments are made before making tax payments. So, the tax rate is
applied to a lower amount of earnings (known as Earnings Before Tax EBT) and
this results in a lower cash outflow for the company. This tax deductible feature
is not present for equity financing as dividends are made after tax payments from
the Net Income line.
7. A bond price will converge to its par value known as pull to par as it reaches
maturity but the share price will tend to have an upward drift, primarily owing to
inflation.

Open University of Mauritius - Financial Theory & Practice

57

6.4 BOND PRICING


(a) The Pricing Mechanism
Pricing a bond is no different to pricing any other security (e.g. equity). It simply relates
to the time value of money and simple discounting.

Similar to the previous example, consider the $1000 bond with semi-annual coupon
rate of 4% maturing in 2 years.
The additional information required is the Required Rate of Return.
Let this be 5% (annualised).

Time
Coupons Principal
(months)
($)
($)
0
6
12
18
24

0
20
20
20
20

Total Cash
Flow ($)

0
0
0
0
1000

0
20
20
20
1020

Discount
Factor

0
1/(1+0.025)=
1/((1+0.025)^2)=
1/((1+0.025)^3)=
1/((1+0.025)^4)=

Discounted
Cash Flow ($)

0.000
0.976
0.952
0.929
0.906
Bond Price

0.00
19.51
19.04
18.57
924.07
981.19

As illustrated above, the cash flows are discounted at the required rate of return at each
individual point in time to back out the bond price.
This Required Rate of Return is called the Yield To Maturity (YTM) and the next
section of Unit will further elaborate on the factors that determine the YTM to be
plugged in the pricing formula.
Bond prices and YTM have an inverse relationship. If the YTM increases by 100 basis
points, i.e. 1%, the bond price will decrease by a certain amount as per the pricing
mechanism above.

(b) Bond Price Dynamics


As mentioned previously, bond pricing is inherently different from that of equity pricing.
The reason behind this difference in dynamics is that, with passage of time, the maturity
of the bond decreases, the amount of coupons in the future decreases and the discount
factor applied to the cash flows changes throughout time. The bond becomes a different
animal one day to the next as a result of its fixed life span.

58

Open University of Mauritius - Financial Theory & Practice

The 4 intuitive bond price volatility theorems are as follows.


1.

Passage of Time The Pull to Par Factor

For the purposes of discussion, it is important to differentiate between par bonds,


premium bonds and discount bonds.
A par bond is one which is trading at the face value $1000. This happens when the
coupon rate and the YTM are equal.
A premium bond is one which is trading above its face value of $1000 and occurs when
the coupon rate is above the YTM.
A discount bond is when the coupon rate is below the YTM and is trading below the
face value.
i) Par Bond
If interest rates do not change and the par bond approaches maturity, there will be no
change in its price: it will continue to sell at par value.
ii) Discount Bond
As a discount bond moves towards maturity, its price will increase even if interest
rates do not change. This can be explained as follows: 1) as the bond moves toward
the maturity there are fewer coupons to be paid and the present value of the coupon
decreases. However: 2) the present value of the par value increases as the time goes by,
as the discount factor applied to calculate its present value becomes smaller as the time
passes. Combining 1) and 2): for the discount bond, the increase in the present value of
the par value is greater than the decrease in the present value of coupons, resulting in the
overall price increase. Graphically, this can be presented as:
Price

Par Value

Market
Price


Time to Maturity
0
iii) Premium Bond
The price of a premium bond decreases as the bond moves towards maturity and there
are no changes in interest rates. The reason for this is similar as for the discount bond.
In particular, 1) the present value of coupons decreases with time and 2) the present
value of the par value increases for the same reasons as before. However, in the case
of a premium bond, the increase in the present value of par is smaller than the decrease
in the present value of coupons (because coupons for the premium bonds can be quite
large, larger than the interest rate), causing an overall decrease in the price of a bond.
Graphically, this can be presented as follows:

Open University of Mauritius - Financial Theory & Practice

59

Price
Market
Price
Par Value



Time to Maturity
0
As it can be seen from the above diagrams, as the bond time to maturity approaches
zero, i.e. the bond is closer to expiry, the market price of the bond approaches par value.
2. Coupon Rate
The lower the coupon rate, the higher the price volatility of a bond. This is because the
price is determined more by the final par value in the discounted cash flow calculation.
For a similar change in YTM, the denominator will fluctuate more for this final payment
than for the coupons to be received in the near future as this final discount rate is raised
to a higher power.
This is the reason that bonds that pay no coupons (known as zero coupon bonds) have
higher volatility.
3. Maturity
Considering 2 bonds with similar characteristics except for their maturities, the bond
with longer maturity will display a higher volatility. Mathematically, the logic behind
this result is that a larger proportion of the cash flows have to be received in a more
distant future. As a result, for a similar change in YTM, the discount factor will change
more for those corresponding cash flows that occur later on.
4. The Level of the YTM
All else equal, a bond with higher yield will have lower volatility than a bond with lower
yield. This is shown graphically below.
Price
The bond price YTM relationship, as mentioned before, is an inverse and, in addition,
a non-linear one. This is evident from the changing gradient of the graph above as
we move across the x-axis and the polynomial equation of the discounted pricing
calculation.
As the gradient takes on larger absolute values, i.e. as the yield decreases, this implies
that the percentage change in price to % change in YTM increases.
Yield
So, the smaller the YTM becomes, the larger the price volatility. This relates to the
concept of duration that will be discussed further in the Unit.

60

Open University of Mauritius - Financial Theory & Practice

6.5 BOND RISKINESS - DURATION & CONVEXITY


I) DURATION

As explained in the section above, there are many determinants that affect the value of
a bond. Depending on the characteristics of the bond, taking into account the above 4
factors individually can lead to ambiguous results as to which bond is the more volatile.
So, a more sophisticated approach is required to measure bond price volatility.
The most commonly used measure is duration.
The strict definition of duration, as developed by MaCaulay, is total weighted average
time to recovery of the payments and principal in relation to the current market price of
the bond, the weights being the present value of the cash flows.
When deriving it, duration takes into account the coupon rate, the YTM and the time to
maturity in a single number. The unit of duration is years.

(a) Conventional Bonds


The equation of MaCaulay duration is given as follows:
1
P

D=

1C
(1 + y)

2C
+
(1 + y)2

nC
(1 + y)n

nF
(1 + y)n

A more compact way of writing it is:


D=

1 n
txPVt
P t-1

where,
PVt =

C
(1 + y)t

t = 1,2 ....., n - 1

C+F
(1 + y)n

t = 1,2 ....., n - 1

and
PVn =

A more intuitive definition of duration is the measure of the sensitivity of bond price
to a change in interest rates. Hence, the alternative way to understand bond volatility is
Modified Duration and is defined as:
MD =

D
1+y

so that the percentage change in the price of the bond is


dD
= (MD)dy
P

In words,
Percentage change in bond price = - Modified Duration x

Change in basis points


100

The modified duration can be viewed as a multiplier. A bond with a modified duration
of 4, for example, experiences approximately a 4% change in price for every 100 basis
point (i.e. 1%) change in yield.

Open University of Mauritius - Financial Theory & Practice

61

For example, if the interest rates rise by 100bps (1%)


Percent change in bond price = - 4.0 x

100
= - 4.00%
100

If the interest rates fall by 100bps:


Percent change in bond price = - 4.0 x

-100
= 4.00%
100

It is important to tie together the 4 theorems of bond price volatility and duration as to
how each of the 4 factors affect duration. With the duration formula in mind
All else equal,
i) A bond with lower coupon rate will have higher duration as the present value of
cash flows is lower and in turn, the market price of the bond, P, is lower.
ii) A bond with lower yield will have higher duration as decreases in the yield will lead
to higher present value of cash flows i.e the terms in the bracket.
iii) A bond with longer time to maturity, i.e. larger n in the equation above, will exhibit
larger variations in the denominator (1+y)n than a bond with lower n (shorter time
to maturity).
It can be concluded that the 4 theorems and duration match up in terms of measuring
the bond price volatility.

(b) Special Cases of Duration Calculation:

There are special cases that duration of certain bonds collapses to smaller formulas.
For example,
Duration of zero coupon bonds = D = n
We can also show that there is an upper limit in the value of duration by considering an
undated bond, i.e. that even though undated bonds do not have time to maturity, they do
have a duration. An undated bond is a bond whose coupon payments are made at regular
intervals but the principal will never be repaid technically. (N.B. such bonds usually
have options attached to them such that principal repayment does occur. Discussions are
not within the scope of the syllabus)
Duration of Undated bonds = D =

1+y
y

II) CONVEXITY

In spite of its sophistication, there are some problems associated with duration especially
for large changes in interest rates.

62

Open University of Mauritius - Financial Theory & Practice

As it has been mentioned before, the bond price YTM relationship is a non-linear
one. However, the duration assumes a linear relationship between the two. From a
practitioners perspective, this might not be totally problematic for small changes in
YTM e.g. in the range of 25bps to 50bps; duration is still an appropriate measure and is
rarely the case that yields move by more than this amount in one instance.
However, for larger changes in YTM, another measure known as convexity is required
to fill the gap to accurately measure price sensitivity.
Convexity is defined as the second derivative of bond price with respect to interest rate.
The formula for convexity is given by the following:
C=

1
P

d2P
dy2


Combined with the modified duration formula, the percentage change in bond price is
given by:
dP
1
- MD (y1 - y0) +
C(y1 - y0)2
P
2


where the first part of the right hand side of the formula is the movement as explained
by the modified duration.
It is important to note that modified duration always works in the favour of the investor
holding the bond! When the interest rate falls, the modified duration model understates
the increase in bond price. On the flipside, when interest rates increase, the model
overstates the losses made on the bond position.
In addition, from the graph and the equation, convexity is always positive as it needs to
increase the price of the bond such that the true value of the bond is obtained.

6.6 YIELD TO MATURITY


(a) Definition and Decomposition

So far, we have looked at the cash flows and price of the bond. However, it is crucial to
understand the discount rate that needs to be applied to the cash flows.
The yield to maturity is defined as the average weighted yield if:
The bond is held to maturity
Coupons are paid on time and reinvested at the YTM rate.
The YTM is only an approximation of the future bond return as it is known at which rate
the coupons will actually be reinvested in the future. It is the interest rate that makes the
present value of the cash flow equal to the bond price and is subjective.
It can be decomposed into several factors. Intuitively, for any type of security, the higher
the risk associated, the higher the discount rate that will be used to discount the future
cash flows of that security. The same logic applies to bonds.

Open University of Mauritius - Financial Theory & Practice

63

Using Government bonds as an example, the discount rate used will be made up of:
(i) The base rate of the country. Typically, this nominal rate is determined by the real
interest rate and the inflation rate. However, in troubled times, this relationship can
will not hold true as Central Banks shift their focus away from inflation targeting
towards stable GDP growth as is the case today. (0.5% in UK, 0.25% in the US)
(ii) The country default premium measuring the issuing country ability to repay its
debt e.g. Greece has a high sovereign default premium associated to its bonds.
(iii) 
Unexpected inflation risk premium is the incremental yield added to Government
bonds to compensate for the fact that despite inflation rate being usually built in the
nominal rate, there is a risk of inflation overshooting nonetheless. Hence, the term
unexpected is used.
(iv) Premium related to price risk/interest rate risk for bonds having higher durations.
(v) Premium related to reinvestment risk because if interest rates fall, the coupons
are reinvested at a lower rate than might have been expected.
There are other sources of risk that an investor will need to be compensated for such as
liquidity premium (to compensate the investor for his inability to unwind his positions
quickly at fair value), event risk, which relate to natural catastrophes and the likes and
foreign exchange risk for investors having positions denominated in currencies other
than their home currency.
Taking the US Treasury bond and the UK Government bond as examples, the largest risk that
needs to be rewarded in the form of the required rate of return will be driven by the nominal
base rate as the other factors tend to be fairly negligible as a result of the developed nature of
the financial system within these countries and the low probability of sovereign default.
In the case of corporate bonds, the above risks hold true as the companies that issue the
bond are domiciled in the country in question. However, in addition, investors need to
take into account corporate credit/default risk.
Credit risk is an important factor in the YTM estimation and needs to be carefully
assessed. It is mainly driven by business deterioration and unfavourable movements in
industry or macroeconomic factors.
Many financial institutions employ credit analysts to determine the repayment ability
of companies. Very often, credit ratings are used to supplement their analysis. Credit
rating agencies such as Moodys and Standard & Poor provide opinions on the future
ability and legal obligation of an issuer to make timely payments of principal and
interest on a specific fixed income security many corporations and Government units
around the globe. (Moodys)
The following table shows the ratings assigned to entities.
Description

Moodys

S&P

Highest quality
High quality

Aaa
Aa

AAA
AA

Strong payment capacity


Adequate payment capacity
Likely to fulfill obligation
High risk obligations

A
Baa
Ba
B

A
BBB
BB
B

Vulnerable to default

Caa

CCC

Investment Grade

Junk Bond
High Yield
Non-investment Grade

Empirically, a large price drop is observed when the corporations are downgraded to
non-investment grade the reason being that pension funds, which are large investors
in bond markets, are not allowed to hold bonds in that category.
64

Open University of Mauritius - Financial Theory & Practice

(b) The Yield Curve

The yield curve is the depiction of the yields of similar bonds (UK Gilts/ Corporate
bonds issued by Microsoft) with varying time to maturities. Investors use the yield
curve as a reference point for forecasting interest rates and pricing bonds.
As shown in the diagrams below, yield curves generally have an upward slope.

UK Gilts Yield Curve November 2012

The 3 mains explanations of the slope of the yield curve are:


i) The pure expectations hypothesis holds that the slope of the yield curve reflects
only the investors expectations of future short term interest rates. In good economic
times, the interest rate is expected to rise to match a rise in expected inflation and
thus, investors demand a higher return as compensation. This explains why the
yield curve has an upward slope. However, the yield curve is a leading indicator of
recessions when it inverts as it reflects the expectations of Central Banks to lower
interest rates to shore up the economy and relax any inflation targeting regime.
ii) The liquidity preference theory also assumes that longer-term bonds require an
iadditional premium for having their capital tied for longer periods of time and for
the higher durations prevalent at the long-end of the yield curve.

Open University of Mauritius - Financial Theory & Practice

65

iii) The market segmentation theory purports that most bond investors in the market
have short to medium term investment horizons. So, in order to lure these investors
to invest in bonds with longer maturities, additional premium is required.

6.7 PRACTITIONERS PERSPECTIVE


With so much to chew on from the theoretical aspect of bonds, it is important to keep
abreast of the main uses of these instruments.

i)

Corporations and Leverage

Many companies use debt on their balance sheet to fund their projects because of the tax
deductibility aspect that leads to reduced cash outflow in the form tax savings.
In addition, leverage can significantly boost the Return on Equity as shown below.

From the diagram above, instead of investing only $100m and obtain a cash inflow of
$200m in year n, the company uses $500m ($400m of additional debt raised), receive a
cash inflow of $1bn and pay off the principal amount of $400 raised and a small interest
amount out of the $600m accruing to the shareholders. Effectively, the boosted the
return to the shareholders by a scale of 3 compared to the previous capital structure.
However, in such levered capital structure, it is of paramount importance for the
company to be able to pay off the interest and debt principal in timely manner. If not,
the firm will default on its debt obligations.

ii) Leveraged Buy-Outs (LBO) Private Equity Funds

Private equity funds comprise of sophisticated investors that raise debt in order to
fund the purchase of all the equity stake of a public company. If successful, the debt
is put on the previously-public company and the asset of the company is pledged as
collateral. Essentially, such funds put in a small amount of their own money to buy-out
the company outright from the secondary market and own the company thanks to the
debt raised in the name of the company they acquire.
There a few companies that can sustain the typical leverage of 70% to 80% loaded in the
firm. Such companies should have strong, stable cash flows, leading market positions,
significant growth opportunities, strong asset base and proven management team.
LBO funds earn a typical return of 20% - 50% on their investment by paying down the
debt from the large cash flows generated by the company and growing the revenues.
The investment horizon is typically 7 years, after which the company is floated in the
secondary market once the debt to asset ratio reaches the norm in the industry.
66

Open University of Mauritius - Financial Theory & Practice

A good example of an LBO transaction is the acquisition of Manchester United by


the Glazer family, despite not being a fund per se, in 2005. The football team satisfies
all the criteria of an LBO target company. The debt put on the balance sheet has
been reduced tremendously since 2005 and the main focus, in addition to winning
silverwares, cashing in on match day revenues and television broadcast income, has
been an aggressive growth in their commercial revenue (which has grown by 80% from
2009) and increasing their reach in Asia.
It is foreseeable that the company will be fully floated in the next 5-7 years once most
debt has been paid and top-line revenue has increased sufficiently. The Glazer family
might be looking at an IRR of above 50% if their strategy comes to fruition.

iii)

Bond Trading Strategies


Bond trading is not as popular as equity trading because of the lower liquidity associated
with debt markets and most trading being done over the counter.
In spite of this, trading strategies have been devised for debt instruments. An intuitive
one would be buy long-term debt with high duration in anticipation of interest rates
falling and the price of the security increasing a larger price increase as compared to
the shorter-term counterparts.
More sophisticated trading strategies have been devised e.g. the hugely popular butterfly
trades, which has been adopted since the mid-80. The mechanism of this trading strategy
is beyond the curriculum as it comprises a long-short position on similar group bonds
with different maturity. As opposed to the above mentioned long-only strategy, this one
aims to maintain a target duration/risk level and does not require cash upfront. The sole
aim of the trader is to increase the yield of portfolio of bonds while tracking an index
against which his performance is measured.
Another widely used strategy is the long-short yield spread trades, where the trader
takes a view on shifts in specific portions of the yield curve.

iv)

Bond Funds

Bond is an attractive investment to common people seeking a regular, riskless stream of


income together with the lower capital risk volatility associated with equity investments.
Nervertheless, few corporate bonds tend to be bought directly by retail investors. The
typical minimum investment of 10,000 precludes all but the wealthiest from building
diversified portfolios. Equally, it is often impossible for retail investors to buy at launch
of an issue, and with most debt currently trading above par, those who buy the bond
at par and keep it towards maturity risk a capital loss. In reality, most retail investors
will invest in corporate bonds via corporate bond fund route. In addition, bonds are
traded over-the-counter and pre-emptive fees have to be paid to brokers to complete
transactions. Very often, detailed analysis is required as firms usually issue bonds with
varying coupons, covenants and maturities that may be trading above or below par.
Therefore, corporate bonds market is dominated by institutional investors such as
pension funds and bond funds such PIMCO that provide shares of bond funds for those
interested with a fixed income exposure.

Open University of Mauritius - Financial Theory & Practice

67

6.8 SUMMARY
This unit represents the building blocks for mastering the analysis, valuation and
management of fixed income securities. In recent years, there has been significant
growth in the fixed income markets of other countries as borrowers have shifted from
funding via banks loans to the issuance of fixed income securities. That is a trend that is
expected to continue. In the next chapter, we will shift our focus to the risk and return
concept and the importance of diversification in this world of uncertainty.

6.9 TUTORIALs
Question 1

Microsoft has issued a 10% Annual Coupon paying Bond maturing on the 20th March
2015. The settlement date is the 20th March 2012 and is sold in denominations of USD
1,000. The yield to maturity is given as 8% and the current market price of the bond is
USD 970. Discuss whether you should buy this bond.

Question 2
You are provided with the following information:
Face value: Rs1000
Maturity: 3 years
Yield: 6.90%
Coupon: 7.5%, semi annual
Settlement date is the same as the coupn date.
Determine the price of the bond.

Question 3

What does the yield curve represent and discuss how the slope should evolve with
reference to the theory underlying the yield curve.

Question 4

Discuss the new trends that are emerging in global bond markets

6.10 SUGGESTED READING


Bodie, Zvi; Kane, Alex; Marcus, Alan J., Investments, Eight Edition

68

Open University of Mauritius - Financial Theory & Practice

RISK, RETURN
AND DIVERSIFICATION

Unit Structure
7.0 Overview
7.1 Learning Objectives
7.2 Introduction
7.3 Measures of Risk and Return
7.4 Risk of the Investment
7.5 Portfolio of Assets
7.6 The Concept of Minimum Variance Frontier
7.7 Summary
7.8 Tutorial
7.9 Suggested Reading

Open University of Mauritius - Financial Theory & Practice

69

7.0 OVERVIEW
It is important to be able to understand that different assets pay off differently in different
states and these states have a certain probability of occurring. As a result of this inherent
riskiness in the cash flows, investors should be able to assess the expected returns and
volatility of assets.
The concept of portfolio management will be introduced and it will demonstrated
why diversification is considered as the only free lunch in finance, especially when
considered on a risk-return plane.

7.1 LEARNING OBJECTIVES


By the end of this Unit, you should be able to do the following:
1. Calculate expected returns and standard deviation of individual assets.
2. Calculate expected returns and standard deviation of a portfolio of 2 assets.
3. Calculate the covariance and correlation between assets.
4. Understand how diversification relates to correlation.
5. Have a notion of the minimum variance opportunity set and what implies in
terms of Sharpe Ratio.

7.2 Introduction
Why is it that the US T Bills provide investors a lower return than Microsoft but have
always been one of the most traded assets across the globe? Is it not the case that the
primary aim of an investor is to earn the highest return possible? The answer is no.
By and large, investors are risk averse and require compensation for the risk they take.
It follows that, from the risk-return relationship, if an asset is riskier, the returns
expected by the investor will be higher. We can interpret that the higher average returns
of Microsoft, or the stock market in general, is to compensate investors for the higher
risk of the investment as compared to the much less riskier T Bill instruments.

7.3 Measures of Risk and Return


Different securities have different initial prices, pay different dividends and sell for
different future share prices. To standardize their performance, their returns should be
calculated.
Return is the percentage increase in value of an investment per amount initially invested
in the security initially.
When a security is risky, there are many different return outcomes that may occur and
these return outcomes have different probability. This information can be summarized
in terms of a probability distribution. As a result of this distribution, the expected
return is the weighted average of all the returns, where the weights are the probabilities.
N

E(R) = piRi
i=1

70

Open University of Mauritius - Financial Theory & Practice

Given the following table about non-dividend paying stock A, the expected return at the
end of 1 year is given as per the calculation below.
Current Share
Price (Rs)

100

Probability,
p (%)

Expected Share Price


in one year (Rs)

Expected
Return, R (%)

25%

140

40%

50%

110

10%

25%

80

-20%

From its current market price of Rs100, the share can either move up or down since
the future outcome remains uncertain. Therefore, a probability of the expected future
outcome (price) is attached to it. Hence, the share has a 25% probability that it will
increase to Rs140, a 50% probability that it will move to Rs110 and a 25% probability
that it will decrease to Rs80.
The expected return is calculated as follows:
Expected Return = (0.25*40% + 0.5*10% + -0.2*25%) = 10%

7.4 Risk of the investment


The two common measures of the risk of a probability are its variance and standard
deviation.
The variance is the expected square deviation from the mean and the standard deviation
is the square root of the variance.
N

var (R) = E[(R1 - E(R))2] = pi(Ri - E (R))2


i=1

(R) = var(R)
The variance is a measure of the spread of the return distribution is. In the case of a riskfree asset, the return is guaranteed; there is no uncertainty about the return. Hence, the
variance of such assets (e.g. US T-Bills) is zero.
Applying the above formula to calculate the variance of the stock A yields:
Variance = 25%*(-0.2-0.1)2 + 50%* (0.1-0.1)2 + 25%* (0.4-0.1)2 = 0.045 = 4.5%
Standard deviation = Variance = 4.5% = 2.12%
Standard deviation is very often referred to as volatility.

Open University of Mauritius - Financial Theory & Practice

71

7.5 Portfolio of assets


The previous sub-sections of this Unit dealt with the expected return and variance of a
single asset. The risk of any stock is made up of two components: a diversifiable and
a non-diversifiable risk component. The diversifiable or unsystematic risks are risks
which are specific to that stock, while the non-diversifiable or systematic risks are
risks associated with economy wide variations. The latter risks cannot be eliminated.
However, investors can eliminate the unsystematic risks by holding a fully diversified
portfolio of assets. As propounded by Harry Markowitz in 1952, investors are able to do
so by holding a portfolio of stocks that do not move exactly together. We will elaborate
on diversification later on, and consider a portfolio of two assets.
Assume 2 risky assets X and Y in a portfolio with weights a% and b% respectively.
The expected return of the portfolio Rp is:
E (Rp ) = aE(Rx) + bE(Ry)
The variance of the portfolio is given by:
var (Rp) = a2 var(Rx ) + b2 var(Ry ) + 2ab cov (Rx, Ry)
In the case of portfolios, since the prices do not move identically, some of the risk
is averaged out in the portfolio. The amount of risk that is eliminated in a portfolio
depends to the degree the stocks face common risks and how their prices move together.
As an example, consider 2 airline company stocks. They tend to move together at the
same time, the portfolio will have risks only slightly lower than that of the individual
stocks. The diversification phenomenon does not kick in. However, if an airline stock
and an oil company stock are combined, and since these typically move in opposite
directions, some of the risk is cancelled out. The portfolio is much less risky.
Covariance and correlation are thus important concepts to understand how diversification
works.
Diversification, covariance and correlation
Usually, stock prices and dividends fluctuate owing to 2 types of risks:
1. 
Firm-specific/Idiosyncratic/Diversifiable risk is news relating to the company
itself e.g. a firm announcing that it missed analyst earnings estimates by a substantial
amount.
2. 
Market-wide/Systematic /Non-diversifiable risk is news about the economy as
a whole e.g. risk that GDP growth is negative or hyperinflation.
Fluctuations that are due news relating to the individual companies themselves can be
averaged out when combined in a portfolio. For instance, good news about company X
will be balanced out with company Y. As such, a minimum of 10 stocks is required to
diversify away all idiosyncratic risk. The only risk left will be owing to systematic risk.
So, the total risk of the portfolio/stock is given by the sum of the unique and systematic
risks and is measured by the variance/standard deviation.
The amount of firm-specific risk that can be diversified will depend on covariance
between the pairs of stocks.

72

Open University of Mauritius - Financial Theory & Practice

Covariance is the expected product of the deviations of 2 returns from the mean. It is
given by the following formula.
Cov (Rx, Ry) = E[(Rx E[ Rx]) (Ry E[Ry ])]
Intuitively, if 2 stocks move together, their returns will tend to be above or below average
at the same time and the covariance will be positive and vice versa.
Correlation is the covariance divided the product of the standard deviations of the
stocks as per the following formula, and it ensures the sign is from -1 to +1.
Cov (Rx, Ry) = Cov (Rx,Ry) / (x. y)
As opposed to covariance, correlation is standardized and a better measure of degree
of linear co-movement. The table below indicates the correlation that exists between
the three instruments: S&P 500, U.S. Long Term Corporate Bonds and MSCI
Emerging Markets
Correlation Matrix
of Returns

S&P 500

U.S. Long-Term
Corporate Bonds

MSCI Emerging
Markets

S&P 500

1.00

0.25

0.45

U.S. Long-Term
Corporate Bonds

0.25

1.00

0.20

MSCI Emerging Markets

0.45

0.20

1.00

As long as security returns are not perfectly correlated, diversification benefits are
possible. Furthermore, the smaller the correlation between security returns, the greater
the costs of not diversifying.
A correlation of 0 (uncorrelated variable) indicates an absence of any linear (straightline) relationship between the variables. Increasingly positive correlation indicates
an increasingly strong positive linear relationship (up to 1, which indicates a perfect
linear relationship). Increasingly negative correlation indicates an increasingly strong
negative (inverse) linear relationship (down to -1, which indicates a perfect inverse
linear relationship).
Example:
Given the following data, find the expected return, covariance, correlation and
variance of a portfolio consisting stock A and B given only the following table and
the weight of each stock being 50%.
Year

Stock Returns
Stock X

Stock Y

2003

21%

9%

2004

30%

21%

2005

7%

7%

2006

-5%

-2%

2007

-2%

-5%

2008

9%

30%

Open University of Mauritius - Financial Theory & Practice

73

Applying the formulae from before for individual stocks X and Y, the following answers
are found:
E(Ri)

10.0%

10.0%

Variance

0.018

0.018

Volatility

13.4%

13.4%

In order to compute the covariance and correlation, the following table should be
drawn up.

Now, we proceed to calculate the portfolio expected return and variance.


E (Rp ) = aE(Rx) + bE(Ry)
Portfolio Expected Returns = 50%*10 + 50%*10 = 10%.
Given, var (Rp) = a2 var (Rx ) + b2 var (Ry ) + 2ab cov (Rx, Ry)
Portfolio Variance = 0.52 * 0.018 + 0.52 * 0.018 + 2 * 0.5 * 0.5* 0.0093 = 0.01365
Portfolio Volatility = 11.7%
From the example above, it can be determined that the 2 stocks have exactly the same
expected return and volatilities. However, when combined together in a portfolio in
equal amounts, the expected return remains the same but the volatility of the portfolio is
11.7% as compared to 13.4% from before.
The volatility/variance denotes the total risk of the portfolio. One important performance
measure of assets is the Sharpe Ratio and is defined as the return of the asset above the
risk-free rate divided by the standard deviation of asset.
Sharpe Ratio =

74

Asset Return - Risk Free rate


Standard Deviation of Asset

Open University of Mauritius - Financial Theory & Practice

Given a risk-free rate of 1%,


Sharpe Ratio X = (0.1-0.01)/0.134 = 0.671
Sharpe Ratio Y = (0.1-0.01)/0.134 = 0.671
Sharpe Ratio P = (0.1-0.01)/0.117 = 0.770
The Sharpe Ratio is a standardized measure of performance as it indicates the expected
return per unit of total risk taken. The higher the ratio, the better the asset.

7.6 The Concept of Minimum


Variance Frontier
In the example of portfolio of stocks X and Y, the weight was randomly selected to be
50% and the volatility reduced from 13.4% to 11.7%. For the same 2 assets, there might
be a combination of weights that gives the investor a larger Sharpe Ratio i.e. a better
return for each unit of risk.
Lets consider the case of stocks Alpha and Beta with the following characteristics.
Alpha: Expected Return: 26%

Volatility: 50%
Beta: Expected Return: 6%

Volatility: 25%
Correlation of 0.
If we invest 40% in A and 60% in B, the expected return of the portfolio is 14% and
the variance is given by:
Expected Return = 0.42*0.52 + 0.62*0.252 + 2(0.4)(0.6)(0)(0.5)(0.25) = 0.0625
The volatility is 25%.
If the same procedure is repeated by varying the weights in the portfolio, we will obtain
duets of expected return and volatility and the result is tabulated below.
Portfolio Weights
Alpha

Beta

Portfolio Expected
Return (%)

Portfolio
Volatility (%)

100%

0%

26.0

50.0

80%
60%
40%
20%

20%
40%
60%
80%

22.0
18.0
14.0
10.0

40.3
31.6
25.0
22.3

0%

100%

6.0

25.0

The minimum variance frontier/ minimum variance opportunity set is defined as


the set of all portfolios of risky assets that yield the minimum variance for a given
rate of expected return.

Open University of Mauritius - Financial Theory & Practice

75

The part of the line below the 20% Alpha and 80% Beta weight consists of inefficient
portfolios the reason being that for the same volatility, a better portfolio closer to
the (0.4,0.6) weight allocation can be found to give investors higher returns for the
same risk.
The portfolio lying on the efficient set and that is tangential to the line drawn from the
risk free asset (lying on the vertical axis with coordinates (0,1%) is called the efficient
portfolio. The gradient of this line is equivalent to the Sharpe Ratio and is maximum
at this point.
Given the 2 assets, this portfolio is the best among all other combinations from a riskretun perspective.

E(Rp)

For different levels of correlation, the frontier will change as shown in the graph below

Impact of Correlation between Assets

A
C

rxy=-1
rxy=+1

For a given asset allocation, correlation has no impact on the expected return as
the correlation is not used in the portfolio expected return calculation. However,
the volatility will change.
When assets are perfectly positively correlated (correlation of +1), the set of portfolios
will be along a straight line and there is no diversification benefit.
When the correlation is less than 1, volatility of the portfolio is reduced due to
diversification and the minimum variance frontier curve bends to the left. This reduction
in volatility is greater as the correlation decreases.

76

Open University of Mauritius - Financial Theory & Practice

O=(Rp)

If an asset which perfectly uncorrelated to another, i.e. correlation of -1, an optimal


weight combination would enable a return above the riskless rate (e.g. US T-Bill) at no
risk as illustrated above.
In the previous sections, we have considered a portfolio consisting of risky assets only.
However, investors can also hold a combination of risky and risk-free assets. A risk-free
asset is one which pays a risk-free rate and has zero standard deviation.
The earlier figure can be combined to show the different possible portfolios consisting
of a risk-free security and risky securities. The Capital Allocation Line (CAL) shows
the manner in which capital can be allocated between the risk-free and risky securities.
It is the line that connects all portfolios that can be formed using the two classes
of assets. Assuming that the investors have three feasible combinations of risky
portfolios (P, B and M) on the efficient frontier, we draw three CALs from the risk-free
rate to these three portfolios of risky assets.
Return
Capital Market Line(CML)
Q
R

N
O

Capital Allocation Lines(CALs)

P
L

Risk,c

Source: Financial Management by Pandey


The best possible CAL is the one which is tangent to the efficient set of risky assets.
It shows the risk-return trade off when the market is in equilibrium. The Capital
Market Line (CML) shows this efficient set of risk-free and risky securities in market
equilibrium.

Open University of Mauritius - Financial Theory & Practice

77

7.7 Tutorials
Question 1
Probability

End of period
Share Price

End of period
Dividends

10 %

19.00

1.00

20 %

20.50

2.00

40 %

22.00

3.00

20 %

26.00

4.00

10 %

35.00

5.00

If the current share price of the stock is Rs25.00, find the expected return and the
standard deviation of the asset.

Question 2
A pension fund manager is considering 2 mutual funds and a Government bond for
investment purposes. The probability distribution of the fund returns are shown as
follows.
Expected Return

Standard Deviation

Stock Fund (S)

20%

30%

Bond Fund (B)

12%

15%

The correlation has been found to be 0.10.


(i) Starting with 100% invested in S and O% in B, compute the expected return
and standard deviation of the 4 portfolios in increments in weight of 20%.
(ii) Draw the minimum frontier opportunity set from the values of expected returns
and volatility.
(ii) Indicate where the optimal portfolio is on the frontier.
(iv) Calculate the Sharpe Ratio for each portfolio. Which one is the best?

Question 3
The following information is given for two risky assets (an Equity Fund and a Debt
Fund):
Securities

Expected Return

Standard Deviation

Equity Fund (E)

0.4

0.25

Debt Fund (D)

0.10

0.13

The correlation between the fund returns is -0.10.


(a) Given that an individual decides to invest 55% of his wealth in Equity Fund
and 45% in Debt Fund, calculate the expected return and standard deviation of
the individuals portfolio.
(b) What are the investment proportions in the minimum-variance portfolio of the two
risky funds?
(c) Calculate the expected return and risk of the minimum variance portfolio?

78

Open University of Mauritius - Financial Theory & Practice

Question 4
The following information is given for three risky assets:
Securities
X
Y
Z

Expected Return
0.25
0.35
0.15

Standard Deviation
0.15
0.23
0.20

The correlation between X and Y is - 0.10.


The correlation between X and Z is - 0.33.
The correlation between Z and Y is + 0.15.
Given that an individual decides to invest 25% in X, 20% in Y and the remaining in Z,
calculate the expected return and standard deviation of the individuals portfolio.

Question 5

You are given below assets X and Y returns along with their associated probabilities:
Probability
0.2
0.2
0.2
0.2
0.2

Asset X:
Returns in %
10
8
20
6
-1

Asset Y:
Returns in %
-2
13
3
25
8

1) Calculate the expected return of Asset X and Asset Y.


2) Calculate the variance and standard deviation of asset X and asset Y.
3) Given that an individual decides to equally invest between asset X and Y, calculate
the expected return of the individuals portfolio.
4) Calculate the individuals portfolio variance and standard deviation.
5) Explain why the portfolio variance is lower than the variance of asset X or asset Y.

7.8 SUGGESTED READINGs


Brealey, Richard A.; Myers, Stewart C., Principles of Corporate Finance, Latest Edition
Pike, Richard; Neale, Bill, Corporate and Finance Investment Decisions and Strategies,
Second Edition
Bodie, Zvi; Kane, Alex; Marcus, Alan J., Investments, Eight Edition

Open University of Mauritius - Financial Theory & Practice

79

80

Open University of Mauritius - Financial Theory & Practice

Open University of Mauritius - Financial Theory & Practice

81

82

Open University of Mauritius - Financial Theory & Practice

Assignment
In todays fast evolving world, Mauritian Hospitality Groups rested on past
laurels and lost out. The shift in client base and requirements is proving to be
a lasting phenomenon with both the local and regional operating environment
becoming increasingly competitive. If the hotel industry is to remain sustainable
in the long-term, it will need to re-engineer its modus operandi and reduce debt
and the Government will need to realize that clientele diversification cannot
happen without new aerial routes
In light of the above statement, analyse the hotel industry, with a thorough research
and analysis of two or more major hotel groups in Mauritius.
Indicative Marking Scheme
- Introduction (10 Marks)
- Analysis of the tourism industry (SWOT analysis) (30 Marks)
- Analysis of two or more hotels: (30 Marks per Hotel Group)

Background of hotel activities and structure (5 Marks)

Analysis of the company (SWOT analysis) (5 Marks)

Performance of the hotel and its management for period 2005-2013 (5 Marks)

Analysis of the groups share price performance and explanation thereof
(5 Marks)

The way forward for the company (5 Marks)

Any additional information (5 Marks)
Note:
1. If the student has opted to analyse for example 4 hotel groups, 60% of the marks
will be divided by four and therefore, the student will have a greater opportunity
to earn higher marks.
2. The assignment will be initially marked on 100 and will then be scaled back to 30.

Open University of Mauritius - Financial Theory & Practice

83

84

Open University of Mauritius - Financial Theory & Practice

SOLUTIONS MANUAL

FINANCIAL THEORY AND PRACTICE


Unit 1: The Financial System
Question 1

He can diversify his portfolio by buying shares in mutual funds or unit trusts. By doing
so, he will benefit from the advantages of diversification whilst keeping his transactions
costs low since he will not be buying many individual shares.

Question 2

(a) Brokers: They connect buyers and sellers. Trades can only be completed if the
brokers find a buyer for each seller and vice-versa. Brokers work on commission.
Market makers: when securities are sold to an investor, market makers buy them at
a given price and try simultaneously or subsequently to sell them at a higher price.
Their earnings are thus the difference between the sell price and the buy price.
(b) Higher, because the risk is higher

Question 3

(a) Saving money for future consumption


(b) To exchange assets for immediate and future deliveries
(c) To manage risks
(d) Price discovery of assets at very low costs, thereby aiding liquidity of financial
products trading in different markets

Question 4

(a) Primary markets new issues of a security; secondary markets subsequent trading
of the shares.
(b) Discussion of the role of investment bankers in the initial public offering process.
(c) Role of brokers and dealers in carrying out trades in primary and secondary markets.
(d) Importance of secondary markets after a share has been issued in the primary market
(e) The aim of the examples is to make the students check out local and international
markets and have a pulse of financial markets. Concrete examples of the stock will
differ across time, but the main aim of this exercise is to make them differentiate
between newly issued stocks and those that are already trading on secondary markets.

Question 5

Discussion of the role of banks as intermediaries between companies and the services
provided by banks:
- Access to equity markets;
- Access to bond markets;
- Mergers and acquisitions;
- Asset management;
- Amongst others.

Open University of Mauritius - Financial Theory & Practice

85

Unit 2: Capital Markets


Question 1

Initial public offerings or IPOs are stocks issued by a formerly privately owned company
that is going public, that is, selling stock to the public for the first time. Seasoned equity
offerings are offered by companies that already have floated equity. For examples, a
sale by IBM of new shares would constitute a seasoned new issue.
Investment bankers normally manage the issue of new securities to the investing
community. Once the Financial and Services Commission has given its approval, the
investment bankers can distribute a prospectus and organize road shows to market the
stock and the company. These road shows are important in that it helps to generate
interest about the company and provide financial and non-financial information about
the firm to investors. The latter can also formulate the price at which they would like to
purchase the stock of the company. These indications of interest are referred to as book
and the process of polling potential investors is referred to as bookbuilding.
Use the case of a recently issued stock e.g. Facebook. Discussion should be centred
around where initial public offering price, the banks that were involved in the process,
the marketing process, the performance of the stock since its listing

Question 2

Differences should be with respect to:


- The type of contract (standardized or tailor made);
- The presence/absence of an intermediary;
- The different types of position;
- Maturity date;
- Value of contract;
- Flexibility of contract terms;
(Note: Student is expected to make some further research on these different types
of markets)

Question 3

Money markets and bond markets cannot be used interchangeably.


The money market is a sub-sector of the fixed income market. It consists of very
short term debt securities that usually are highly liquid. These securities trade in large
denominations and are out of reach of individual investors. Examples of instruments
traded on money markets include Treasury Bills, Certificates of Deposits, Commercial
Paper, amongst others. Treasury Bills are most liquid and marketable of all money
market instruments. Investors buy bills at a discount from the stated maturity value.
At the bills maturity, the holder receives from the government a payment equal to the
face value of the bill. The difference constitutes the investors gain. T-Bills are issued
with initial maturities of 28, 91, 182 days. Individuals and institutional investors can
purchase T-Bills directly at auction or on the secondary market from a government
securities dealer. A certificate of deposit or CD is a time deposit with the bank. Time
deposits may not be withdrawn on demand. The bank pays interest and principal to the
depositor only at the end of the fixed of the CD. Commercial papers are short-term
unsecured debt notes that are issued by large and well-known companies. Commercial
paper maturities range up to 270 days and are considered to be fairly safe assets since
a firms condition presumably can be monitored and predicted over a term as short as 1
month.
On the other hand, the bond market comprises longer term debt instruments other than
those that trade on the money market. This market includes Treasury notes and bonds,
corporate bonds, municipal bonds, amongst others. Most of these instruments promise
86

Open University of Mauritius - Financial Theory & Practice

either a fixed stream of income or a stream of income that is determined according to a


specific formula. It is common practice to refer to these instruments as debt instruments
or bonds. Treasury notes and bonds are issued by the Government and are virtually
considered as being risk free i.e. there is no risk that the Government will default on
repayment. Treasury notes have a maturity of up to 10 years while Treasury bonds have
a maturity greater than 10 years. These debt instruments pay annual or semi-annual
coupons over the life of the bond, with final repayment at maturity. Corporate bonds
are issued when firms borrow money directly from the public. These bonds are listed on
an exchange and are typically similar to Treasury notes and bonds in that they pay annual
or semi-annual coupons over the life of the bond and return the face value to the holder
of the bond at maturity. Municipal bonds are issued by state and local governments.
They are similar to treasury securities exempt that their interest income is exempt from
tax. The interest income is also exempt from state and local taxation in the issuing state.
Capital gains tax is however payable when the bonds mature or are sold prior to their
maturity dates.
(Note: Encourage the students to research further instruments traded on these two
different markets and differentiate between the different types of financial instruments)

Unit 3: The Time Value of Money


Question 1

The first annual payment is effected on 31 December 2012.


1400000 = A x

1
(1+0.15)10
0.15


Find the value of A.
The first annual payment is effected on 1 January 2012.
1400,000 = A + A x

1
(1+0.15)10-1
0.15


Find the value of A.

Question 2

Size of fund- FV = PV (1 + r)n = 30,000 (1.05)10


EAR = ( 1 + 0.05)2 - 1

Question 3


(i)
FV = PV (1 + r)n = 2,000 (1.10)5
(ii)
FV = PV (1 + r)n = 2,000 (1.10)5*2 = 2,000 (1.10)10
(iii)
FV = PV (1 + r)n = 2,000 (1.10)5*4 = 2,000 (1.10)20
Open University of Mauritius - Financial Theory & Practice

87

Unit 4: Capital Investment Appraisal


Question 1
(a) Calculation of ARR
Project A
Depreciation = 200 / 4 = 50m
1
2
90
85
(50)
(50)
40
35

Year
Future CF
Depreciation
Profit

3
80
(50)
30

Average Profit = (40+35+30+30) / 4 = 33.75


ARR = (33.75/200) x 100% = 16.9%

Project B
Depreciation = 200 / 4 = 50m
1
2
75
70
(50)
(50)
25
20
Average Profit = (25+20+15+15) / 4 = 18.75
ARR = (18.75/200) x 100% = 9.4%

Year
Future CF
Depreciation
Profit

4
80
(50)
30

3
65
(50)
15

4
65
(50)
15

(b) Calculation of Payback

Year
0
1
2

Cash Flow
-200
90
85

Project A
Cummulative Cash Flow
-200
-110
-25

3
4

80
80

55
135

Time
0
1
2
(25/80) x 1 year or 12 Months = 0.31
or approximately 4 months
-

Year
0
1
2

Cash Flow
-200
75
70

Project B
Cummulative Cash Flow
-200
-125
-55

Time
0
1
2

3
4

65
65

10
75

(55/65) x 1 year or 12 month = 0.85


years or approximately 11 months
-

Therefore, the payback period for project A is 2 years and 4 months.


Therefore the Payback period for project B is 2 years and 11 months.
(c) NPV of project A = +11.05; NPV of project B = -26.36.
(d) IRR of project A = 25.04%; IRR of project B =14.5%.

88

Open University of Mauritius - Financial Theory & Practice

Question 2
(a)
Year
Revenue
Maintenance
Costs
Scrap
Net Cash
Flow

0
(155,000)

1
200,000

2
210,000

3
220,500

4
231,525

5
243,101

6
-

(10,000)

(11,500)

(13,225)

(15,209)

(17,490)

20,000

(155,000)

190,000

198,500

207,275

216,316

225,611

20,000

(b) At a 15% discount rate, the Net Present Value = +541,093.05


(c) The IRR of the project is 124.47%
(d) Payback = 0.815 years.

Question 3

(a) NPV (Project 1) = +3,730.25


(b) NPV (Project 2) = +8,528.79

(a) IRR (Project 1) = 17.37%


(b) IRR (Project 2) = 20.46%

Unit 5: Share Valuation, Risk and Return


Question 1
(a)
D2008 = D2004 x (1 + g)4

2.28 = 1.35 x (1 + g) 4
Therefore, g = 14%

Gordon Growth Model:



Value of share = D1/(k-g) = (2.28 x 1.14) / (0.19-0.14) = $51.98

(b) Based on the above calculation, the share is observed to be undervalued on


the market and should therefore be bought since the intrinsic value of the share
is greater than the market price of $39.91.
(c) Value of share = (2.28 x 1.13) / (0.19 0.13) = $ 42.94
(d) Value of share = (2.28 x 1.13) / (0.20 0.13) = $36.8

Question 2

Value of share five years from now = (4.00 x 1.06) / (0.10 0.06) = $106
Present value of share = 106 x (1/1.105) = $65.82

Open University of Mauritius - Financial Theory & Practice

89

Question 3
(a)

USD

2008

EBIT
Tax

530
150

+Depreciation & Amortisation

481

- Capital Expenditure

248

- Change on working capital

189

Free Cash Flow

424

(b)






Total Present Value of FCF = 359


Terminal value:
FCF = 30
g=0
WACC = 8%
Terminal Value = 375
PV of Terminal Value = 375/1.083 = 298
Hence, Enterprise value = 657

Unit 6: Fixed Income Securities


Question 1

Value of bond = USD 1,051.54


Since the value of the bond is greater than USD 1000, the bought ought to be bought.

Question 2

Price of bond = Rs 1010.60

Question 3

The yield curve is the depiction of the yields of similar bonds (UK Gilts/ Corporate
bonds issued by Microsoft) with varying time to maturities. Investors use the yield
curve as a reference point for forecasting interest rates and pricing bonds. The 3 mains
explanations of the slope of the yield curve are:
i)
The pure expectations hypothesis holds that the slope of the yield curve
reflects only the investors expectations of future short term interest rates.
In good economic times, the interest rate is expected to rise to match a
rise in expected inflation and thus, investors demand a higher return as
compensation. This explains why the yield curve has an upward slope.
However, the yield curve is a leading indicator of recessions when it inverts
as it reflects the expectations of Central Banks to lower interest rates to
shore up the economy and relax any inflation targeting regime.
ii)

90

The liquidity preference theory also assumes that longer-term bonds require
an additional premium for having their capital tied for longer periods of time
and for the higher durations prevalent at the long-end of the yield curve.
Open University of Mauritius - Financial Theory & Practice

iii)

The market segmentation theory purports that most bond investors in the
market have short to medium term investment horizons. So, in order to
lure these investors to invest in bonds with longer maturities, additional
premium is required.

Question 4

The emerging trends are:


-
Corporations and leverage
-
Leveraged Buy-Outs (LBOs) Private Equity
-
Bond Trading Strategies
-
Bond Funds

Unit 7: Risk, Return and Diversification


Question 3

(a) Expected Return=0.55*0.4+0.45*0.1


Variance= 0.55^2(0.25)^2+0.45^2*0.13)^2+2(0.55)(0.45)(0.25)(0.13)(-0.1)

Risk= sqrt(variance)

(b) Use this formula to find the weight for the min variance portfolio for equity fund (E)
a* =

2d - red e d
2e + 2d - 2red e d

Weight of Debt fund= 1-a*

(c) Based on the weights in part (b), use the following formula;
E(Rp)=We E(Re ) + Wd E(Rd )

p = W2e 2e+W2d 2d + 2WeWd edTed

Question 4

Expected Return= 0.25*0.25+0.2*0.35+0.55*0.15


Using the following formula to find variance and then take square root for risk;

Open University of Mauritius - Financial Theory & Practice

91

Question 5

(a)
E(Rx)= 8.6%
E(Ry)=9.4%
(b)
X
0.004624
0.068

Variance
Standard Deviation

(c)

E(R) = (0.5x8) + (0.5 x 9.4)

(d)

Variance= 0.5*0.004624 + 0.5*0.008584 +2*0.5*0.5*Cov(x,y)


Where Cov (x,y) =
Rx E(Rx)
0.014
-0.006
0.114
-0.026
-0.096

(e)

92

Y
0.008584
0.0925

Ry E(Ry)
-0.114
0.036
-0.064
0.156
-0.014
Total

COV(XY)
-0.0016
-0.00022
-0.0073
-0.00406
0.001344
-0.01182

 he covariance is negative which imply that assets returns tend to move in the
T
opposite direction such that a loss in asset X will be compensated by a gain in
Asset Y or vice versa.

Open University of Mauritius - Financial Theory & Practice

You might also like